Крок 1 - Медицина 2020 1 день (буклет)

1 / 180
Хворому призначена ендоскопія 12-палої кишки. В результаті виявлено запалення великого дуоденального сосочка і порушення виділення жовчі в просвіт кишки. У якому відділі 12-палої кишки виявлені порушення? The patient is scheduled for endoscopy of the duodenum. As a result, inflammation of the large duodenal papilla and violation of the secretion of bile into the lumen of the intestine were detected. In which part of the duodenum were violations detected?

Горизонтальна частина Horizontal part

Висхідна частина Eastern part

Низхідна частина Descending part

Верхня частина Top

Цибулина Onion

2 / 180
При аналізі ЕКГ встановлено: ритм синусовий, число передсердних комплексів перевищує число шлуночкових комплексів; прогресуюче подовження інтервалу Р-Q від комплексу до комплексу; випадання окремих шлуночкових комплексів, через що після зубця Р йде довга пауза; зубці Р та комплекси QRST без змін. Назвіть тип порушення серцевого ритму: When analyzing the ECG, it was established: sinus rhythm, the number of atrial complexes exceeds the number of ventricular complexes; progressive lengthening of the P-Q interval from complex to complex; loss of individual ventricular complexes, due to that there is a long pause after the P wave; P waves and QRST complexes are unchanged. Name the type of heart rhythm disturbance:

Неповна атріовентрикулярна блокада І ступеня I degree incomplete atrioventricular block

Синоатріальна блокада Sinoatrial block

Повна атріовентрикулярна блокада Complete atrioventricular block

Неповна атріовентрикулярна блокада III ступеня Incomplete atrioventricular block III degree

Неповна атріовентрикулярна блокада II ступеня Incomplete atrioventricular block II degree

3 / 180
Чоловік 55-ти років, що скаржиться на біль в ділянці нирок, надійшов в лікарню. В ході ультразвукового обстеження пацієнта виявлено наявність ниркових каменів. Наявність в сечі якої з наведених речовин є найімовірнішою причиною утворення каменів у цього пацієнта? A 55-year-old man, complaining of pain in the kidney area, came to the hospital. During an ultrasound examination of the patient, the presence of kidney stones was detected. The presence of which of the following in the urine which of the following substances is the most likely cause of stone formation in this patient?

Білірубін Bilirubin

Сечова кислота Uric acid

Уробілін Urobilin

Креатинін Creatinine

Білівердин Biliverdin

4 / 180
У 70-річної людини швидкість поширення пульсової хвилі виявилася суттєво вище, ніж у 25-річної. Причиною цього є зниження: In a 70-year-old person, the pulse wave propagation speed was significantly higher than in a 25-year-old person. The reason for this is a decrease:

Серцевого викиду Cardiac output

Артеріального тиску Blood pressure

Швидкості кровотоку Blood flow rates

Частоти серцевих скорочень Heart rate

Еластичності судинної стінки Vascular wall elasticity

5 / 180
Чоловік внаслідок транспортної аварії втратив багато крові, свідомість затьмарена, низький кров’яний тиск. При цьому у нього компенсаторно активується ренін-ангіотензинова система, що веде до: A man lost a lot of blood as a result of a traffic accident, his consciousness is clouded, his blood pressure is low. At the same time, he has a compensatory activation of the renin-angiotensin system, which leads to:

Посилення серцевих скорочень Increasing heart rate

Гіперпродукції вазопресинуАпо В48 Hyperproduction of vasopressin Apo B48

Підвищення згортання крові Increased blood coagulation

Гіперпродукції альдостерону Hyperproduction of aldosterone

Посилення еритропоезу Enhanced erythropoiesis

6 / 180
При дослідженні людини у вертикальній позі встановлено, що в альвеолах верхівок легень парціальний тиск кисню становить 140 мм рт.ст. Причиною цього є те, що у цих відділах легень: When examining a person in a vertical position, it was established that in the alveoli of the tops of the lungs, the partial pressure of oxygen is 140 mm Hg. The reason for this is that in these parts of the lungs :

Вентиляція відсутня No ventilation

Вентиляція переважає над перфузією Ventilation prevails over perfusion

Перфузія переважає над вентиляцією Perfusion prevails over ventilation

Перфузія та вентиляція врівноважені Perfusion and ventilation balanced

7 / 180
Хворий протягом останнього року став відзначати підвищену втомлюваність, загальну слабкість. Аналіз крові: еритроцити -4,1•1012/л, Нb-119 г/л, к.п.- 0,87, лейкоцити - 57•109/л, лейкоформула: Ю- 0, П- 0, С- 9%, Е- 0, Б- 0, лімфобласти - 2%, пролімфоцити - 5%, лімфоцити - 81%, М- 3%, тромбоцити - 160•109/л. В мазку: нормохромія, велика кількість тіней Боткіна-Гумпрехта. Про яку патологію системи крові свідчить дана гемограма? During the last year, the patient began to note increased fatigue, general weakness. Blood analysis: erythrocytes -4.1•1012/l, Hb-119 g/l, k.p.- 0.87, leukocytes - 57•109/l, leukoformula: Y- 0, P- 0, C- 9%, E- 0 . -Gumprecht. What pathology of the blood system does this hemogram indicate?

Гострий лімфобластний лейкоз Acute lymphoblastic leukemia

Хронічний мієлолейкоз Chronic myelogenous leukemia

Гострий мієлобластний лейкоз Acute myeloblastic leukemia

Хронічний монолейкоз Chronic monoleukosis

Хронічний лімфолейкоз Chronic lymphocytic leukemia

8 / 180
У хворого з синдромом Іценка- Кушинга спостерігаються стійка гіперглікемія та глюкозурія. Синтез та секреція якого гормону підвищені у цього хворого? A patient with Itsenko-Cushing syndrome has persistent hyperglycemia and glucosuria. Which hormone's synthesis and secretion are increased in this patient?

Альдостерон Aldosterone

Кортизол Cortisol

Глюкагон Glucagon

Тироксин Thyroxine

Адреналін Adrenaline

9 / 180
Для лікування стрептодермії лікар призначив хворому мазь, що містить антибіотик групи тетрацикліну, і рекомендував обмежити перебування на сонці. Чим небезпечна інсоляція? To treat streptoderma, the doctor prescribed an ointment containing an antibiotic of the tetracycline group to the patient, and recommended limiting exposure to the sun. What is the danger of insolation?

Розвитком стійкості збудника до антибіотика The development of resistance of the pathogen to the antibiotic

Підвищенням токсичності тетрацикліну Increasing tetracycline toxicity

Генералізацією процесу Process generalization

Зниженням активності тетрацикліну Decreased activity of tetracycline

Розвитком фотосенсибілізації Development of photosensitization

10 / 180
На аутопсії тіла жінки, що хворіла на хронічну дизентерію, в ході гістологічного дослідження внутрішніх органів в стромі та паренхімі міокарда, нирок, в слизовій оболонці шлунка та в сполучній тканині легень виявлені аморфні відкладення фіолетового кольору, що дають позитивну реакцію за Коссом. Яке ускладнення розвинулося у хворої'? At the autopsy of the body of a woman suffering from chronic dysentery, during the histological examination of internal organs in the stroma and parenchyma of the myocardium, kidneys, in the mucous membrane of the stomach and in the connective tissue purple amorphous deposits were found in the lungs, giving a positive reaction according to Koss. What complication developed in the patient'?

Метаболічне звапніння Metabolic calcification

Гіаліноз Hyalinosis

Метастатичне звапніння Metastatic calcification

Амілоїдоз Amyloidosis

Дистрофічне звапніння Dystrophic calcification

11 / 180
У 12-річного хлопчика в сечі виявлено високий вміст усіх амінокислот аліфатичного ряду. При цьому відзначена найвища екскреція цистину та цистеїну. Крім того, УЗД нирок показало наявність у них каменів. Виберіть можливу патологію: In a 12-year-old boy, a high content of all amino acids of the aliphatic series was detected in the urine. At the same time, the highest excretion of cystine and cysteine was noted. In addition, ultrasound of the kidneys showed the presence of stones. Select a possible pathology:

Фенілкетонурія Phenylketonuria

Хвороба Хартнупа Hartnup's disease

Цистинурія Cystinuria

Цистит Cystitis

Алкаптонурія Alkaptonuria

12 / 180
При мікроскопічному дослідженні біоптату нирки виявлено вогнища, в центрі яких розташовані зернисті еозинофільні маси, оточені інфільтратом з лімфоцитів, епітеліоїдних клітин та поодиноких клітин Пирогова-Лангханса. Виберіть патологічний процес, що найповніше відповідає зазначеним змінам: During a microscopic examination of a kidney biopsy, foci were found, in the center of which there are granular eosinophilic masses, surrounded by an infiltrate of lymphocytes, epithelioid cells, and single Pirogov-Langhans cells. Select a pathological process , which most fully corresponds to the specified changes:

Коагуляційний некроз Coagulation necrosis

Казеозний некроз Caseous necrosis

Альтеративне запалення Alterative inflammation

Гранулематозне запалення Granulomatous inflammation

Проліферація та диференціювання макрофагів Proliferation and differentiation of macrophages

13 / 180
У дитини 8-ми років через 5 днів після контакту з хворим на вітряну віспу з’явилося нездужання, підвищилася температура, незабаром з’явилися характерні висипи. Який термін найточніше характеризує період, що передував появі перших ознак захворювання? 5 days after contact with a chickenpox patient, an 8-year-old child became unwell, the temperature rose, and characteristic rashes soon appeared. What is the term most accurately characterizes the period preceding the appearance of the first signs of the disease?

Контагіозний Contagious

Інфекційний Infectious

Неспецифічний Non-specific

Інкубаційний Incubation

Латентний Latent

14 / 180
Хворому, після радіоактивного опромінення, лікар рекомендував збільшити в раціоні вміст рослинних олій - джерела полієнових жирних кислот. Назвіть кислоту, що містить три подвійних зв’язки: After radioactive exposure, the doctor recommended increasing the content of vegetable oils in the diet - sources of polyene fatty acids. Name the acid containing three double bonds:

Ліноленова Linoline

Арахідонова Arakhidonova

Лауринова Laurinova

Олеїнова Oleinova

Міристинова Miristinova

15 / 180
Хвора 55-ти років з гострим нападом печінкової кольки надійшла в гастроентерологічне відділення. Об’єктивно: температура тіла - 38°С, склери, слизові та шкіра іктеричні, сеча темна, кал безбарвний. Скарги на свербіння шкіри. Яка причина жовтяниці у цієї хворої? A 55-year-old patient with an acute attack of hepatic colic was admitted to the gastroenterology department. Objectively: body temperature - 38°С, sclera, mucous membranes and skin are icteric, the urine is dark, the feces are colorless. Complaints about itching of the skin. What is the cause of jaundice in this patient?

Обтурація жовчних проток Bile duct obstruction

Посилений розпад еритроцитів Enhanced breakdown of erythrocytes

Порушення ліпідного обміну Lipid metabolism disorder

Деструкція гепатоцитів Destruction of hepatocytes

Тривале вживання продуктів, багатих на каротин Long-term use of foods rich in carotene

16 / 180
Хвора 57-ми років для лікування гіпертонічної хвороби тривалий час приймала анаприлін. Побічні ефекти спонукали пацієнтку відмовитись від прийому препарату, що призвело до розвитку гіпертонічного кризу і нападу стенокардії. Як називається ускладнення, яке виникло? A 57-year-old patient took anaprilin for a long time to treat hypertension. The side effects prompted the patient to stop taking the drug, which led to the development of a hypertensive crisis and an attack of angina pectoris. What is the name of the complication that occurred?

Лікарська залежність Drug addiction

Синдром відміни Withdrawal Syndrome

Тахіфілаксія Tachyphylaxis

Звикання Addiction

Сенсибілізація Sensitization

17 / 180
У хворої після операції видалення ракової пухлини молочної залози та регіонарних лімфовузлів розвинувся набряк руки, який був пов’язаний з лімфатичною недостатністю. Який це вид лімфатичної недостатності за механізмом виникнення? After surgery to remove a cancerous tumor of the breast and regional lymph nodes, the patient developed swelling of the hand, which was associated with lymphatic insufficiency. What type of lymphatic insufficiency is this by the mechanism of occurrence ?

Акінетична Akinetic

Спастична Spastic

Резорбційна Resorbable

Механічна Mechanical

Динамічна Dynamic

18 / 180
У жінки, що тривало приймала антибіотики з приводу кишкової інфекції, розвинулось ускладнення з боку слизової порожнини рота у вигляді запального процесу і білого нальоту, у якому під час бактеріологічного дослідження були виявлені дріжджеподібні грибки Сапdіda аlbісаns. Який з перерахованих препаратів показаний для лікування цього ускладнення? A woman who had been taking antibiotics for an intestinal infection for a long time developed a complication from the mucous membrane of the mouth in the form of an inflammatory process and a white plaque, in which during a bacteriological examination yeast-like fungi Sapdida albisans were detected. Which of the listed drugs is indicated for the treatment of this complication?

Флуконазол Fluconazole

Фуразолідон Furazolidone

Тетрациклін Tetracycline

Бісептол Biseptol

Поліміксин Polymixin

19 / 180
В результаті експресії окремих компонентів геному клітини зародка набувають характерних для них морфологічних, біохімічних та функціональних особливостей. Яку назву має цей процес? As a result of the expression of individual components of the genome, germ cells acquire their characteristic morphological, biochemical and functional features. What is the name of this process?

Рецепція Reception

Диференціювання Differentiation

Індукція Induction

Капацитація Capacitance

Детермінація Determination

20 / 180
Жінка 54-х років звернулася до лікаря зі скаргами на непереносимість курячих яєць, що з’явилася нещодавно. Антигістамінні препарати, призначені лікарем, дещо покращували стан хворої. Які антитіла могли сприяти розвитку цієї реакції? A 54-year-old woman turned to the doctor with complaints of intolerance to chicken eggs, which appeared recently. The antihistamines prescribed by the doctor somewhat improved the patient's condition. Which antibodies could contribute to the development of this reaction?

ІgD IgD

ІgМ IgM

IgA IgA

IgG IgG

IgE IgE

21 / 180
В результаті інфікування рани після видалення зуба у хворого виникло її нагноєння. Для очищення рани, як один із засобів, рекомендовано полоскання рота гіпертонічним розчином NaCl. Який процес лежить в основі цього способу очищення? As a result of infection of the wound after tooth extraction, the patient developed suppuration. To clean the wound, rinsing the mouth with hypertonic NaCl solution is recommended as one of the means. What process lies in basis of this cleaning method?

Бактерицидна дія Bactericidal action

Гемостатична дія Hemostatic effect

Осмотична активність солі Osmotic activity of salt

Денатурація білка Protein denaturation

Механічне очищення Mechanical cleaning

22 / 180
Під час підйому пішки на 5-й поверх у людини підвищився артеріальний тиск. Причиною є збільшення: While climbing on foot to the 5th floor, a person's blood pressure increased. The reason is the increase:

Об’єму циркулюючої крові Circulating blood volume

Вмісту іонів в плазмі крові Content of ions in blood plasma

Хвилинного об’єму крові Minute volume of blood

В’язкості крові Blood viscosity

Кількості функціонуючих капілярів Numbers of functioning capillaries

23 / 180
На 9-й день після введення протиправцевої сироватки з приводу брудної рани стопи в пацієнта підвищилася температура тіла до 38°С, з’явився біль у суглобах, висип, свербіння шкіри. З боку крові спостерігаються лейкопенія і тромбоцитопенія. Який тип алергічної реакції розвинувся? On the 9th day after the introduction of anti-tetanus serum for a dirty wound on the foot, the patient's body temperature rose to 38°C, pain in the joints appeared, a rash, itching of the skin. Leukopenia and thrombocytopenia are observed from the blood side. What type of allergic reaction has developed?

Стимулюючий Stimulating

Цитотоксичний Cytotoxic

Анафілактичний Anaphylactic

Імунокомплексний Immunocomplex

Клітинно-опосередкований Cell-mediated

24 / 180
Тварині в експерименті перерізали передні корінці п’яти сегментів спинного мозку. Які зміни відбудуться в зоні іннервації? Animals in the experiment cut the front roots of five segments of the spinal cord. What changes will occur in the innervation zone?

Втрата пропріоцептивної чутливості Loss of proprioceptive sensitivity

Втрата рухів Loss of movements

Втрата дотикової чутливості Loss of touch sensitivity

Втрата температурної чутливості Loss of temperature sensitivity

Гіперчутливість Hypersensitivity

25 / 180
У потерпілого травма черепа з порушенням дірчастої пластинки решітчастої кістки. Який нерв може бути ушкоджений? The victim has a skull injury with a violation of the perforated plate of the ethmoid bone. What nerve can be damaged?

N. oрthalmicus N. orthalmicus

N.abducens N.abducens

N. оlfactorius N. olfactorius

N. trосhlіаris N. troshliaris

N.opticus N.opticus

26 / 180
У хворого 50-ти років через місяць після перенесеного інфаркту міокарда розвинулася серцева недостатність. Невідповідність між навантаженням на серце та його здатністю виконувати роботу пов’язана в цьому випадку з: A 50-year-old patient developed heart failure one month after a myocardial infarction. In this case, the discrepancy between the load on the heart and its ability to perform work is associated with :

Підвищеним опором вигнанню крові в аорті Increased resistance to expulsion of blood in the aorta

Ушкодженням міокарда Myocardial damage

Змішаною формою недостатності серця Mixed form of heart failure

Перевантаженням серця Heart overload

Підвищеним опором вигнанню крові в легеневому стовбурі Increased resistance to the expulsion of blood in the pulmonary trunk

27 / 180
У хворого після резекції шлунка розвинулась мегалобластична анемія. Який препарат необхідно призначити хворому? The patient developed megaloblastic anemia after gastric resection. What drug should be prescribed to the patient?

Заліза лактат Iron lactate

Вітамін В6 Vitamin B6

Аскорбінову кислоту Ascorbic acid

Ферковен Ferkoven

Вітамін В12 Vitamin B12

28 / 180
У жінки 62-х років розвинулася катаракта (помутніння кришталику) на тлі цукрового діабету. Вкажіть, який тип модифікації білків має місце при діабетичній катаракті: A 62-year-old woman developed a cataract (clouding of the lens) on the background of diabetes. Indicate what type of protein modification occurs in diabetic cataract:

Фосфорилювання Phosphorylation

Обмежений протеоліз Limited proteolysis

Метилювання Methylation

Глікозилювання Glycosylation

АДФ-рибозилювання ADP-ribosylation

29 / 180
Розділ фармакології, що вивчає всмоктування, розподіл та елімінацію ліків в організмі людини, називається: The branch of pharmacology that studies the absorption, distribution, and elimination of drugs in the human body is called:

Токсикологія Toxicology

Фармакокінетика Pharmacokinetics

Імунофармакологія Immunopharmacology

Фармакодинаміка Pharmacodynamics

Фармакогенетика Pharmacogenetics

30 / 180
При пульмонологічному обстеженні виникла необхідність визначити частину повітря, яка обмінюється в легенях за один дихальний цикл. Цей показник називається: During a pulmonological examination, it became necessary to determine the part of air that is exchanged in the lungs during one respiratory cycle. This indicator is called:

Хвилинна легенева вентиляція Minute pulmonary ventilation

Функціональна остаточна ємність Functional final capacity

Об’єм мертвого простору Volume of dead space

Дихальний коефіцієнт Respiratory ratio

Коефіцієнт легеневої вентиляції Coefficient of pulmonary ventilation

31 / 180
Для кращого огляду дна очного яблука лікар закрапав в кон’юнктиву ока пацієнта розчин атропіну. Це призвело до розширення зіниці через блокаду таких мембранних циторецепторів: For a better examination of the fundus of the eyeball, the doctor instilled atropine solution into the conjunctiva of the patient's eye. This led to dilation of the pupil due to the blockade of the following membrane cytoreceptors:

Н2-рецепторів H2-receptors

Бета-адренорецепторів Beta-adrenoceptors

М-холінорецепторів M-cholinergic receptors

Альфа-адренорецепторів Alpha-adrenoceptors

Н-холінорецепторів H-cholinergic receptors

32 / 180
Жінка 38-ми років надійшла в хірургічне відділення з приводу розлитого гнійного перитоніту. Під час операції виявлена перфорація виразки клубової кишки, виразку було ушито. Черевну порожнину дреновано. Через 4 дні хвора померла. На розтині в клубовій кишці наявні виразки в пеєрових бляшках, розташовані уздовж кишки, дно деяких з них має поперечну смугастість. Для якого захворювання характерні такі виразки? A 38-year-old woman was admitted to the surgical department for diffuse purulent peritonitis. During the operation, a perforation of the ileal ulcer was detected, the ulcer was sutured. The abdominal cavity was drained. Through The patient died 4 days later. At the autopsy in the ileum, there are ulcers in Peyer's plaques, located along the intestine, the bottom of some of them has transverse striations. For what disease are such ulcers characteristic?

Пухлини тонкої кишки Tumors of the small intestine

Дизентерії Dysentery

Амебіазу кишечника Intestinal amebiasis

Туберкульозу кишечника Intestinal tuberculosis

Черевного тифу Typhoid

33 / 180
Хворий помер від наростаючої легенево-серцевої недостатності. При гістологічному дослідженні виявлено дифузне ураження легенів з інтерстиціальним набряком та інфільтрацією інтер-стиціальної тканини лімфоцитами, макрофагами, плазмоцитами; наявні пневмофіброз та панацинарна емфізема. Поставте діагноз: The patient died of progressive pulmonary heart failure. Histological examination revealed diffuse lung damage with interstitial edema and infiltration of interstitial tissue with lymphocytes, macrophages, plasma cells; pneumofibrosis was present and panacinar emphysema. Diagnose:

Бронхопневмонія Bronchopneumonia

Фіброзивний альвеоліт Fibrosing alveolitis

Ателектаз легенів Atelectasis of the lungs

Бронхіальна астма Bronchial asthma

Хронічний бронхіт Chronic bronchitis

34 / 180
В процесі розвитку у дитини хребет поступово набув два лордози та два кіфози. Це пояснюється розвитком здатності до: During the child's development, the spine gradually acquired two lordosis and two kyphosis. This is explained by the development of the ability to:

Сидіння Seats

Прямоходіння Direct

Плавання Swimming

Лежання Lying

Повзання Crawling

35 / 180
Судово-медичний експерт при розтині тіла 20-річної дівчини встановив смерть внаслідок отруєння ціанідами. Порушення якого процесу, найімовірніше, стало причиною смерті? At the autopsy of the body of a 20-year-old girl, the forensic medical expert found death due to cyanide poisoning. What process violation most likely caused death?

Транспорту амінокислот Transport of amino acids

Тканинного дихання Tissue breathing

Синтезу сечовини Urea synthesis

Транспорту кисню гемоглобіном Oxygen transport by hemoglobin

Синтезу гемоглобіну Hemoglobin synthesis

36 / 180
В лікарню звернувся 65-річний хворий зі скаргами на часте сечовиділення зі слідами крові. При посіві сечі на МПА виросли великі колонії з характерним квітковим запахом, які мали зелено- синій пігмент. В мазку з колоній наявні грамнегативні палички. Який збудник спричинив запальний процес сечостатевої системи? A 65-year-old patient came to the hospital with complaints of frequent urination with traces of blood. When urine was cultured on MPA, large colonies with a characteristic flowery smell grew, which had green- blue pigment. Gram-negative bacilli are present in the colony smear. What pathogen caused the inflammatory process of the genitourinary system?

Proteus vulgaris Proteus vulgaris

Pseudomonas aeruginosa Pseudomonas aeruginosa

Hafnia alvei Hafnia alvei

Кlebsiella ozaenae Klebsiella ozaenae

Escherichia coli Escherichia coli

37 / 180
Чоловік з гострим міокардитом помер від серцево-судинної недостатності. В ході мікроскопічного дослідження внутрішніх органів виявлені: плазморагія, набряк, стази в капілярах, численні крововиливи, а також дистрофічні зміни в паренхімі. Наслідком чого є дані зміни? A man with acute myocarditis died of cardiovascular failure. Microscopic examination of internal organs revealed: plasmarrhagia, edema, stasis in capillaries, numerous hemorrhages, as well as dystrophic changes in the parenchyma. What are the consequences of these changes?

Загальне артеріальне повнокрів’я Total arterial complete blood

ДВ3-синдром DV3 syndrome

Хронічний загальний венозний застій Chronic general venous stasis

Місцеве артеріальне повнокрів’я Local arterial complete blood

Гострий загальний венозний застій Acute general venous stasis

38 / 180
В епідермісі є клітини, що виконують захисну функцію та мають моноцитарний генез. Які це клітини? In the epidermis there are cells that perform a protective function and have a monocytic origin. What are these cells?

Кератиноцити базального шару Keratinocytes of the basal layer

Клітини Лангерганса Langerhans cells

Кератиноцити остистого шару Keratinocytes of the spinous layer

Кератиноцити зернистого шару Keratinocytes of the granular layer

Меланоцити Melanocytes

39 / 180
Чоловік 40-ка років скаржиться на загальну слабкість, головний біль, кашель із виділенням мокротиння, задишку. Після клінічного огляду й обстеження поставлено діагноз пневмонія. Який тип гіпоксії має місце у хворого? A 40-year-old man complains of general weakness, headache, cough with sputum, shortness of breath. After a clinical examination and examination, a diagnosis of pneumonia was made. What type of hypoxia does he have the patient's place?

Гіпоксична Hypoxic

Циркуляторна Circulator

Гемічна Chemical

Респіраторна Respiratory

Тканинна Fabric

40 / 180
У підлітка після перенесеного інфекційного захворювання з’явилася різко виражена аритмія з вкороченням інтервалу R-R під час вдиху і подовженням його під час видиху. Що лежить в основі цього виду аритмії? After an infectious disease, a teenager developed a pronounced arrhythmia with a shortening of the RR interval during inhalation and a lengthening of it during exhalation. What is the basis of this type of arrhythmia ?

Рефлекс Бейнбріджа Bainbridge Reflex

Порушення функції провідності серця Disruption of the conduction function of the heart

Порушення функції збудливості серця Disruption of heart excitability function

Коливання тонусу блукаючого нерва під час акту дихання Vacation of vagus nerve tone during breathing

Порушення скоротливої функції серця Disruption of the contractile function of the heart

41 / 180
Жінці 58-ми років проведене повне видалення матки з придатками, після чого виділення сечі припинилося. При цистоскопії: міхур сечі не містить, з устів сечоводів сеча не надходить. Який відділ сечовидільної системи було ушкоджено в ході операції? A 58-year-old woman underwent a complete removal of the uterus with appendages, after which urine flow stopped. During cystoscopy: the bladder does not contain urine, urine does not flow from the mouths of the ureters. What part of the urinary system was damaged during the operation?

Ureter Ureter

Vesica urinaria Vesica urinaria

Uretra Uretra

Rеп Rep

Pelvis renalis Pelvis renalis

42 / 180
У чоловіка 30-ти років перед операцією визначили групову належність крові. Кров резус-позитивна. Реакцію аглютинації еритроцитів не викликали стандартні сироватки груп 0αβ (І), Аβ (II), Вα (III). Досліджувана кров належить до групи: The blood group of a 30-year-old man was determined before the operation. The blood is Rh-positive. The erythrocyte agglutination reaction was not caused by standard sera of groups 0αβ (I), Aβ ( II), Bα (III). The studied blood belongs to the group:

Вα (III) Bα (III)

Aβ (II) Aβ (II)

АВ (IV) AB (IV)

- -

0αβ (І) 0αβ (And)

43 / 180
У лікарню машиною швидкої допомоги доставлено хворого в стані коми. В анамнезі цукровий діабет. При обстеженні спостерігається шумне прискорене дихання, при якому глибокі вдихи чергуються з посиленими видохами за участю експіраторних м’язів. Яка форма порушення зовнішнього дихання спостерігається? A comatose patient was brought to the hospital by ambulance. He has a history of diabetes. During the examination, noisy rapid breathing is observed, in which deep breaths alternate with increased exhalations with the participation of expiratory muscles. What form of external breathing disorder is observed?

Стенотичне дихання Stenotic breathing

Дихання Чейна-Стокса Cheyne-Stokes Respiration

Дихання Куссмауля Kussmaul breathing

Апнейстичне дихання Apneic breathing

Дихання Біота Biot's Breath

44 / 180
У хворого після черепно-мозкової травми, під час якої була ушкоджена мозочкова ділянка, розвинулися порушення часової та просторової координації рухів. Яка патологія розвинулася у хворого? After a craniocerebral injury, during which the cerebellar area was damaged, the patient developed disturbances in temporal and spatial coordination of movements. What pathology developed in the patient?

Дисметрія Dysmetria

Парез Paresis

Абазія Abaziya

Атаксія Ataxia

Астазія Astasia

45 / 180
Пацієнт 55-ти років звернувся до лікаря зі скаргами на часті судоми. Встановлено, що тривалий час він працює у гарячому цеху в умовах високих температур. Порушення якого виду обміну призвело до цього стану? A 55-year-old patient turned to the doctor with complaints of frequent seizures. It was established that he has been working for a long time in a hot workshop in conditions of high temperatures. Violation of what type of metabolism led to this state?

Вітамінного Vitamin

Водно-сольового Water-salt

Вуглеводного Carbohydrate

Білкового Protein

Ліпідного Lipid

46 / 180
У результаті радіаційного випромінювання були ушкоджені стовбурові гемопоетичні клітини. Утворення яких клітин сполучної тканини буде порушено? Hematopoietic stem cells were damaged as a result of radiation. The formation of which connective tissue cells will be disturbed?

Перицити Pericytes

Макрофаги Macrophages

Меланоцити Melanocytes

Адипоцити Adipocytes

Фібробласти Fibroblasts

47 / 180
У потерпілого пошкоджений м’яз, апоневроз якого утворює пахвинну зв’язку. Назвіть цей м’яз: The victim has a damaged muscle, the aponeurosis of which forms the inguinal ligament. Name this muscle:

Поперечний м’яз живота Transverse abdominis muscle

Пірамідальний м’яз Pyramidalis muscle

Зовнішній косий м’яз живота External oblique muscle

Прямий м’яз живота Recti abdominis

Внутрішній косий м’яз живота Internal oblique muscle

48 / 180
При обстеженні хворого похилого віку виявлено моторну афазію. Де локалізований осередок пошкодження головного мозку? During the examination of an elderly patient, motor aphasia was detected. Where is the localized center of brain damage?

Постцентральна звивина Postcentral gyrus

Звивина Гешля Heschl's gyrus

Прецентральна звивина Precentral gyrus

Кутова звивина Angular gyrus

Центр Брока Broca Center

49 / 180
У сільського жителя на кисті правої руки з’явилася неболюча припухлість, яка набула в центрі чорного кольору. При мікроскопічному дослідженні виявлено великі грампозитивні палички, розташовані ланцюжками. Який мікроорганізм міг спричинити це захворювання? A villager had a painless swelling on his right hand, which turned black in the center. Microscopic examination revealed large gram-positive rods arranged in chains. What kind of microorganism could cause this disease?

Bacillus cereus Bacillus cereus

Clostridium tetani Clostridium tetani

Clostridium botulinum Clostridium botulinum

Mycobacterium tuberculosis Mycobacterium tuberculosis

Bacillus anthracis Bacillus anthracis

50 / 180
Симбіотична теорія пояснює походження еукаріотичних клітин переходом до аеробного дихання. Це відбулося внаслідок проникнення в клітину аеробних бактерій, які в процесі еволюції перетворилися на: The symbiotic theory explains the origin of eukaryotic cells by the transition to aerobic respiration. This occurred as a result of the penetration of aerobic bacteria into the cell, which in the process of evolution turned into:

Мітохондрїї Mitochondria

Лізосоми Lysosomes

Пероксисоми Peroxisomes

Комплекс Гольджі Golgi Complex

Рибосоми Ribosomes

51 / 180
Оглядаючи дитину 6-ти років, лікар помітив на глоткових мигдаликах сірувату плівку, при спробі видалення якої виникла помірна кровотеча. Бактеріоскопія мазків з мигдаликів показала наявність грампозитивних бактерій булавоподібної форми. Які симптоми можуть виникнути у дитини у найближчі дні, якщо не буде проведене специфічне лікування? While examining a 6-year-old child, the doctor noticed a grayish film on the pharyngeal tonsils, which, when trying to remove it, caused moderate bleeding. Bacterioscopy of smears from the tonsils showed the presence of mace-shaped gram-positive bacteria . What symptoms may appear in the child in the coming days, if no specific treatment is carried out?

Хвилеподібна лихоманка Waving fever

Набряк легенів Pulmonary edema

Токсичні ураження серцевого м’язу, печінки, нирок Toxic lesions of heart muscle, liver, kidneys

Дуже сильний нападоподібний кашель Very severe paroxysmal cough

Папульозні висипи на шкірі Papular rashes on the skin

52 / 180
Деякі тяжкі захворювання нирок супроводжуються еритропенією. Який механізм цього явища найімовірніший? Some severe kidney diseases are accompanied by erythropenia. What is the most likely mechanism of this phenomenon?

Підвищене руйнування еритроцитів в селезінці Increased destruction of erythrocytes in the spleen

Аліментарний дефіцит Fе2+ Alimentary deficiency Fe2+

Збільшення діурезу Increased diuresis

Порушення синтезу еритропоетинів Disruption of erythropoietin synthesis

Порушення функціонування печінки Liver dysfunction

53 / 180
У померлого, що понад 20 років працював на шахті з видобутку кам’яного вугілля, при розтині виявлено ущільнені легені сіро-чорного кольору зі значними ділянками новоутвореної сполучної тканини та наявністю великої кількості макрофагів із пігментом чорного кольору в цитоплазмі. Який з перерахованих діагнозів найімовірніший? The deceased, who worked at a coal mine for more than 20 years, had gray-black compacted lungs with significant areas of newly formed connective tissue and by the presence of a large number of macrophages with black pigment in the cytoplasm. Which of the listed diagnoses is the most likely?

Талькоз Talkoz

Антракосилікоз Anthracosilicosis

Силікоантракоз Silicoanthracosis

Антракоз Anthracosis

Сидероз Siderosis

54 / 180
У крові резус-негативної жінки під час вагітності виявлені специфічні білки, здатні руйнувати резус-позитивні еритроцити плода. Як називається цей захисний компонент організму матері? Specific proteins capable of destroying Rh-positive erythrocytes of the fetus were detected in the blood of a Rh-negative woman during pregnancy. What is the name of this protective component of the mother's body?

Антитіло Antibody

Резус-фактор Rhesus Factor

Сироватка Serum

Фактор некрозу пухлини Tumor necrosis factor

Гормон Hormone

55 / 180
Хворий 50-ти років скаржиться на поліурію, спрагу, протягом доби випиває до 15 літрів рідини. При обстеженні виявлено: вміст глюкози крові - 4,8 ммоль/л, сеча безбарвна, відносна щільність - 1,002-1,004, цукор і білок відсутні. Яка імовірна причина поліурії? A 50-year-old patient complains of polyuria, thirst, drinks up to 15 liters of liquid during the day. Examination revealed: blood glucose content - 4.8 mmol/l , urine is colorless, relative density - 1.002-1.004, sugar and protein are absent. What is the probable cause of polyuria?

Надлишок тиреоїдних гормонів Excess thyroid hormones

Дефіцит тиреоїдних гормонів Deficiency of thyroid hormones

Дефіцит вазопресину Vasopressin deficiency

Дефіцит альдостерону Aldosterone deficiency

Надлишок альдостерону Aldosterone excess

56 / 180
У пацієнта перфоративна виразка передньої стінки шлунка. В яке похідне очеревини потрапить вміст шлунка? The patient has a perforating ulcer of the anterior wall of the stomach. Into which derivative of the peritoneum will the contents of the stomach enter?

Лівий брижовий синус Left mesenteric sinus

Чепцева сумка Cap bag

Передшлункова сумка Pregastric pouch

Правий брижовий синус Right mesenteric sinus

Печінкова сумка Liver bag

57 / 180
За умов тривалої інтоксикації тварин тетрахлорметаном було визначене суттєве зниження активності аміноацил- тРНК-синтетаз в гепатоцитах. Який метаболічний процес порушується в цьому випадку? Under the conditions of long-term intoxication of animals with tetrachloromethane, a significant decrease in the activity of aminoacyl-tRNA synthetases in hepatocytes was determined. What metabolic process is disturbed in this case?

Посттранскрипційна модифікація РНК Post-transcriptional modification of RNA

Посттрансляційна модифікація пептидів Post-translational modification of peptides

Реплікація ДНК DNA Replication

Біосинтез білків Biosynthesis of proteins

Транскрипція РНК RNA Transcription

58 / 180
У хворого зі швидко наростаючою внутрішньомозковою гіпертензією діагностована пухлина мозку. Під час операції видалена пухлина тім’яно-скроневої частки, м’якої консистенції, строката на розрізі. Гістологічно пухлина побудована з поліморфних гіперхромних клітин з утворенням псевдорозеток та великої кількості судин, ділянками некрозів і крововиливами. Поставте діагноз: A brain tumor was diagnosed in a patient with rapidly increasing intracerebral hypertension. During the operation, a tumor of the parietal-temporal lobe, soft consistency, variegated on the section, was removed. Histologically the tumor is composed of polymorphic hyperchromic cells with the formation of pseudorosettes and a large number of vessels, areas of necrosis and hemorrhages. Make a diagnosis:

Арахноїдендотеліома Arachnoidendothelioma

Гліобластома Glioblastoma

Астроцитома Astrocytoma

Менінгіома Meningioma

Олігодендрогліома Oligodendroglioma

59 / 180
Експериментатору необхідно якнайшвидше виробити умовний рефлекс у собаки. На базі якого безумовного рефлексу доцільно виробляти умовний? The experimenter needs to develop a conditioned reflex in a dog as soon as possible. On the basis of which unconditioned reflex is it advisable to develop a conditioned one?

Міотатичний Myotatic

Орієнтувальний Indicative

Статевий Gender

Захисний Protective

Травний Digestible

60 / 180
У хворого 60-ти років під час об’єктивного обстеження виявлено набряки на ногах, асцит, збільшення печінки, що свідчить про недостатність кровообігу за правошлуночковим типом. В анамнезі перенесений ревматизм. Одним з основних факторів розвитку набряків є активація системи ренін-ангіотензин- альдостерон, яка є наслідком: During an objective examination, a 60-year-old patient has swelling on the legs, ascites, liver enlargement, which indicates right ventricular circulatory insufficiency. In the anamnesis transferred rheumatism. One of the main factors in the development of edema is the activation of the renin-angiotensin-aldosterone system, which is a consequence of:

Зменшення хвилинного об’єму серця Decreased cardiac output

Ацидозу Acidosis

Поліцитемічної гіперволемїї Polycythemic hypervolemia

Утруднення дифузії речовин Diffusion of substances

Розширення посткапілярних вен Postcapillary venous dilatation

61 / 180
У п’ятимісячної дівчинки виявлено застійні явища у легенях. При обстеженні виявлено зв’язок між висхідною аортою та легеневою артерією, що в нормі спостерігається у деяких земноводних і плазунів. Назвіть цю природжену ваду розвитку: A five-month-old girl was found to have congestion in the lungs. The examination revealed a connection between the ascending aorta and the pulmonary artery, which is normally observed in some amphibians and reptiles . Name this congenital malformation:

Дефект міжпередсердної перегородки Atrial septal defect

Розвиток правої дуги аорти Development of the right aortic arch

Транспозиція магістральних судин Transposition of trunk vessels

Дефект міжшлуночкової перегородки Ventricular septal defect

Незрощення боталової протоки Nonunion of the botal duct

62 / 180
Під час проведення морфологічного дослідження периферичної крові хворого було помічено, що у еритроцитів забарвлена лише периферична частина, а в центрі є незабарвлене прояснення. Кольоровий показник - 0,56. Яка анемія найімовірніша у цього пацієнта? During the morphological examination of the patient's peripheral blood, it was noticed that only the peripheral part of the erythrocytes is stained, and there is a colorless clearing in the center. The color index is 0.56. What anemia is most likely in this patient?

В₁₂ фолієводефіцитна B₁₂ folic acid deficiency

Сидеробластна Syderoblastna

Гемолітична Hemolytic

Апластична Aplastic

Залізодефіцитна Iron deficiency

63 / 180
Під час хірургічної операції виникла необхідність масивного переливання крові. Група крові потерпілого - III (В) Rh(+). Якого донора треба вибрати? During the surgical operation, a massive blood transfusion was required. The victim's blood group is III (B) Rh(+). Which donor should be chosen?

III (В) Rh(-) III (B) Rh(-)

III (В) Rh(+) III (B) Rh(+)

IV (АВ) Rh(-) IV (AB) Rh(-)

І(0)Rh(-) And(0)Rh(-)

II (А) Rh(+) II (А) Rh(+)

64 / 180
У людини внаслідок лікування антибіотиками виник дисбактеріоз товстого кишечника. Яких вітамінів, синтезованих бактеріями в товстому кишечнику, буде менше надходити до організму? Due to antibiotic treatment, a person developed dysbiosis of the large intestine. Which vitamins synthesized by bacteria in the large intestine will be less available to the body?

Аскорбінова кислота Ascorbic acid

Вітамін В Vitamin B

Вітаміни А та Е Vitamins A and E

Вітаміни Р і С Vitamins P and C

Вітамін К та вітаміни групи В Vitamin K and B vitamins

65 / 180
Лікар швидкої допомоги встановив у хворого діагноз гіпертонічний криз. Для усунення цього стану хворому парентерально введено засіб, який крім гіпотензивної має також протисудомну дію, а при ентеральному введенні - проносну і жовчогінну. Назвіть цей препарат: The emergency physician diagnosed the patient with a hypertensive crisis. To eliminate this condition, the patient was administered parenterally a drug that, in addition to hypotensive, also has an anticonvulsant effect, and when administered enterally - a laxative and choleretic. Name this drug:

Магнію сульфат Magnesium sulfate

Пентамін Pentamine

Клофелін Clofelin

Дибазол Dibazol

Натрію нітропрусид Sodium nitroprusside

66 / 180
Жінка 62-х років скаржиться на частий біль грудного відділу хребта, переломи ребер. Лікар припустив мієломну хворобу (плазмоцитому). Який з перерахованих нижче лабораторних показників буде мати найбільше діагностичне значення? A 62-year-old woman complains of frequent pain in the thoracic spine, rib fractures. The doctor suspected myeloma disease (plasmocytoma). Which of the following laboratory indicators will have the most diagnostic value?

Гіпоглобулінемія Hypoglobulinemia

Протеїнурія Proteinuria

Парапротеїнемія Paraproteinemia

Гіпопротеїнемія Hypoproteinemia

Гіперальбумінемія Hyperalbuminemia

67 / 180
У чоловіка, померлого від внутрішньої кровотечі (гемоперитонеум), в печінці субкапсулярно виявлено губчастий вузол темно-червоного кольору розмірами 15x10 см, добре відмежований від навколишньої тканини. Мікроскопічно: тканина вузла складається з великих судинних тонкостінних порожнин, вистелених ендотеліальними клітинами та заповнених рідкою або згорнутою кров’ю. Встановіть вид пухлини: In a man who died of internal bleeding (hemoperitoneum), a dark red spongy nodule measuring 15x10 cm was found in the liver subcapsularly, well separated from the surrounding tissue. Microscopically: the tissue of the node consists of large vascular thin-walled cavities lined with endothelial cells and filled with liquid or coagulated blood. Establish the type of tumor:

Лімфангіома Lymphangioma

Гемангіоперицитома Hemangiopericytoma

Кавернозна гемангіома Cavernous hemangioma

Капілярна гемангіома Capillary hemangioma

Венозна гемангіома Venous hemangioma

68 / 180
Після пошкодження мозку у людини порушене сприйняття зорової інформації. В якому відділі кори сталося пошкодження? After brain damage, a person's perception of visual information is impaired. In which part of the cortex did the damage occur?

Потилична ділянка кори Occipital cortex

Тім’яна ділянка кори Parietal cortex

Передня центральна звивина Anterior central gyrus

Задня центральна звивина Posterior central gyrus

Скронева ділянка кори Temporal cortex

69 / 180
Новонароджений, який з’явився на світ у домашніх умовах без кваліфікованої медичної допомоги, і мати якого не проходила у період вагітності необхідних обстежень, на другу добу доставлений в лікарню з гострим гнійним кон’юнктивітом. При мікроскопії виділень з очей дитини знайдені грамнегативні диплококи, розташовані всередині лейкоцитів та поза клітинами. Який мікроорганізм є найімовірнішим збудником захворювання? A newborn who was born at home without qualified medical care, and whose mother did not undergo the necessary examinations during pregnancy, was taken to the hospital on the second day with acute purulent conjunctivitis. Microscopy of secretions from the child's eyes revealed gram-negative diplococci located inside leukocytes and outside the cells. Which microorganism is the most likely causative agent of the disease?

Сhlamydia trachomatis Chlamydia trachomatis

Соrynebacterіит dірhtheriае Sorynebacteriit dihrtheriae

Neisseria gonorrhoae Neisseria gonorrhoeae

Pseudomonas aeruginosa Pseudomonas aeruginosa

Staphylococcus aureus Staphylococcus aureus

70 / 180
Фізіологи встановили, що кількість еритроцитів у крові залежить від функціонального стану червоного кісткового мозку й тривалості життя еритроцита. Який термін життя еритроцита в периферичній крові в середньому? Physiologists found that the number of erythrocytes in the blood depends on the functional state of the red bone marrow and the life span of the erythrocyte. What is the average life span of an erythrocyte in peripheral blood?

120 діб 120 days

220 діб 220 days

150 діб 150 days

50 діб 50 days

70 діб 70 days

71 / 180
Жінка 35-ти років звернулася до лікаря зі скаргами на дратівливість, тривожність, швидку втомлюваність, безсоння. Для усунення неврозу лікар призначив пацієнтці транквілізатор діазепам. Вкажіть фармакодинамічний ефект діазепаму, що дав можливість застосувати його за цих обставин: A 35-year-old woman consulted a doctor with complaints of irritability, anxiety, rapid fatigue, insomnia. To eliminate neurosis, the doctor prescribed the tranquilizer diazepam to the patient. Specify the pharmacodynamic effect of diazepam , which made it possible to apply it under these circumstances:

Протисудомний Anticonvulsant

Психостимулювальний Psychostimulant

Анксіолітичний Anxiolytic

Міорелаксантний Myorelaxant

Антипсихотичний Antipsychotic

72 / 180
В колективі дошкільної установи одночасно виявили декілька дітей з катаральним запаленням слизової оболонки трахеї та бронхів. За клінічними проявами припущено коклюшну інфекцію. Яким чином, як правило, передається коклюш? Several children with catarrhal inflammation of the mucous membrane of the trachea and bronchi were discovered at the same time in the staff of the preschool institution. According to the clinical manifestations, a whooping cough infection was assumed. How is whooping cough usually transmitted?

Повітряно-краплинним Air-drop

Контактним Contact

Трансмісивним Transmissive

Аліментарним Alimony

Повітряно-пиловим Air-dust

73 / 180
У хворого на тромбофлебіт нижніх кінцівок з’явилися біль в грудній клітці, кровохаркання, наростаюча дихальна недостатність, при явищах якої він помер. На розтині діагностовані множинні інфаркти легенів. Яка найімовірніша причина їх розвитку в цьому випадку? A patient with thrombophlebitis of the lower extremities developed chest pain, hemoptysis, increasing respiratory failure, the symptoms of which caused him to die. At autopsy, multiple lung infarctions were diagnosed. What is the most likely reason for their development in this case?

Тромбоемболія бронхіальних артерій Thromboembolism of bronchial arteries

Тромбоз легеневих вен Thrombosis of pulmonary veins

Тромбоз гілок легеневої артерії Thrombosis of pulmonary artery branches

Тромбоемболія гілок легеневої артерії Thromboembolism of the branches of the pulmonary artery

Тромбоз бронхіальних артерій Thrombosis of bronchial arteries

74 / 180
У хворого видалили щитоподібну залозу, яка була значно збільшена в розмірах, щільно-еластичної консистенції, з горбистою поверхнею. При гістологічному дослідженні в паренхімі залози визначається дифузна лімфо-плазмоцитарна інфільтрація з формуванням лімфоїдних фолікулів з гермінативними центрами, атипія та метаплазія фолікулярного епітелію, вогнища склеротичних змін паренхіми. Який діагноз найімовірніший? The thyroid gland was removed from the patient, which was significantly increased in size, dense-elastic consistency, with a bumpy surface. Histological examination revealed diffuse lympho-plasmacytic in the parenchyma of the gland infiltration with the formation of lymphoid follicles with germinal centers, atypia and metaplasia of the follicular epithelium, foci of sclerotic changes in the parenchyma. What is the most likely diagnosis?

Дифузний тиреотоксичний зоб Diffuse thyrotoxic goiter

Тиреоїдит де Кервена De Quervain's thyroiditis

Вузловий зоб Nodular goiter

Аутоімунний тиреоїдит Autoimmune thyroiditis

Дифузний еутиреоїдний зоб Diffuse euthyroid goiter

75 / 180
У патогенезі розвитку II типу гіперліпопротеїнемії (сімейна гіперхолестеролемія) провідну роль відіграє дефіцит рецепторів до апобілка ЛПНЩ. Назвіть його: In the pathogenesis of the development of type II hyperlipoproteinemia (familial hypercholesterolemia), a deficiency of receptors for LDL apoprotein plays a leading role. Name it:

Апо ВСІ Apo ALL

Апо СІІ Apo SII

Апо А1 Apo A1

Апо В100 Apo B100

Апо В48 Apo B48

76 / 180
Хвороба Куру характеризується тремором і атаксією; при хворобі Крейтцфельдта-Якоба спостерігаються розвиток атаксії і деменції. Доведено, що ці захворювання викликаються: Kuru's disease is characterized by tremors and ataxia; in Creutzfeldt-Jakob disease, ataxia and dementia develop. It has been proven that these diseases are caused by:

Грибами Mushrooms

Повільними вірусами Slow viruses

Бактеріями, позбавленими клітинної стінки Bacteria lacking a cell wall

Пріонами Prions

Токсичними продуктами навколишнього середовища Toxic environmental products

77 / 180
У хворої діагностовано пухлину головки підшлункової залози, порушення венозного відтоку з деяких органів черевної порожнини. Яка венозна судина була здавлена пухлиною? The patient was diagnosed with a tumor of the head of the pancreas, violation of venous outflow from some organs of the abdominal cavity. Which venous vessel was squeezed by the tumor?

Ниркова вена Renal vein

Ліва шлункова вена Left gastric vein

Права шлункова вена Right gastric vein

Ворітна вена печінки Portal vein of the liver

Нижня порожниста вена Inferior vena cava

78 / 180
У дитини спостерігаються невротичні симптоми: запаморочення, слабкість, головний біль, який супроводжується нудотою, болем в правому підребер’ї, частими позивами на дефекацію. При лабораторному дослідженні дуоденального вмісту виявлено грушоподібні най-простіші з двома ядрами, 4-ма парами джгутиків, а у фекаліях овальної форми цисти. Яке захворювання у дитини? The child has neurotic symptoms: dizziness, weakness, headache, which is accompanied by nausea, pain in the right hypochondrium, frequent urges to defecate. During the laboratory examination of the duodenal the contents revealed pear-shaped protozoans with two nuclei, 4 pairs of flagella, and oval-shaped cysts in the feces. What disease does the child have?

Амебіаз Amebiasis

Лямбліоз Giardiasis

Балантидіаз Balantidiasis

Кишковий трихомоноз Intestinal trichomoniasis

Токсоплазмоз Toxoplasmosis

79 / 180
Психологічне дослідження встановило: у людини добра здатність швидко пристосовуватися до нового оточення, добра пам’ять, емоційна стійкість, висока працездатність. Найімовірніше, ця людина: Psychological research established: a person has a good ability to quickly adapt to a new environment, good memory, emotional stability, high work capacity. Most likely, this person:

Сангвінік Sanguine

Флегматик Phlegmatic

Меланхолік Melancholic

Флегматик з елементами меланхоліка Phlegmatic with elements of melancholic

Холерик Choleric

80 / 180
В організмі людини визначено порушення обміну мелатоніну. Це може бути пов’язано з нестачею амінокислоти, з якої мелатонін синтезується. Яка це амінокислота? Impaired melatonin metabolism has been determined in the human body. This may be due to a lack of the amino acid from which melatonin is synthesized. What is this amino acid?

Триптофан Tryptophan

Гістидин Histidine

ДОФА DOFA

Аланін Alanine

Глутамат Glutamate

81 / 180
У хворих із синдромом набутого імунодефіциту (СНІД) різко знижується імунологічна реактивність, що проявляється розвитком хронічних запальних процесів, інфекційних захворювань, пухлинного росту. Клітини якого типу ушкоджує ВІЛ-інфекція, внаслідок чого знижується імунний захист? In patients with acquired immunodeficiency syndrome (AIDS), immunological reactivity is sharply reduced, which is manifested by the development of chronic inflammatory processes, infectious diseases, tumor growth. What type of cells is damaged by HIV- infection, as a result of which the immune defense decreases?

Т-супресори T-suppressors

Т4-хелпери T4-helpers

Т8-ефектори T8-effectors

Природні кілери (NК) Natural killers (NK)

В-лімфоцити B-lymphocytes

82 / 180
При глікогенозі (хворобі Гірке) пригнічується перетворення глюкозо-6- фосфату в глюкозу, що супроводжується порушенням розпаду глікогену в печінці. Дефіцит якого ферменту є причиною цього захворювання? With glycogenosis (Ghirke's disease), the conversion of glucose-6-phosphate into glucose is inhibited, which is accompanied by a violation of the breakdown of glycogen in the liver. What enzyme deficiency is the cause of this disease?

Глікогенфосфорилази Glycogen phosphorylases

Глюкозо-6-фосфатази Glucose-6-phosphatases

Глюкозо-6-фосфатдегідрогенази Glucose-6-phosphate dehydrogenase

Фосфофруктокінази Phosphofructokinase

Фосфоглюкомутази Phosphoglucomutases

83 / 180
У хворого шкіра чутлива до сонячного світла. Назвіть це спадкове захворювання, зумовлене дефектами ферментів системи репарації ДНК: The patient's skin is sensitive to sunlight. Name this hereditary disease caused by defects in enzymes of the DNA repair system:

Хвороба Леша-Ніхана Lesh-Nyhan disease

Порфірія Porphyria

Пігментна ксеродермія Xeroderma pigmentosum

Вітиліго Vitiligo

Альбінізм Albinism

84 / 180
Ряд антибіотиків є специфічними інгібіторами процесу трансляції в мікроорганізмах. Робота яких органел порушується при цьому? A number of antibiotics are specific inhibitors of the translation process in microorganisms. Which organelles are affected by this?

Лізосом Lysosome

Рибосом Ribosome

Мікротрубочок Microtube

Пероксисом Peroxisome

Мітохондрій Mitochondrion

85 / 180
При недостатньому харчуванні часто має місце білкове голодування, яке проявляється зниженням вмісту білка в плазмі крові й розвитком набряків. За рахунок яких білків плазми більшою мірою розвивається зниження онкотичного тиску при цьому? With insufficient nutrition, protein starvation often occurs, which is manifested by a decrease in the content of protein in the blood plasma and the development of edema. Due to which plasma proteins, a decrease in oncotic pressure develops to a greater extent in this?

Фібриногену Fibrinogen

Гамма-глобулінів Gamma globulins

Альбумінів Albumin

Бета-глобулінів Beta-globulins

Альфа-глобулінів Alpha globulins

86 / 180
Для профілактики атеросклерозу, ішемічної хвороби серця та порушень мозкового кровообігу людина повинна одержувати 2-6 г незамінних поліненасичених жирних кислот на добу. Ці кислоти необхідні для синтезу: For the prevention of atherosclerosis, coronary heart disease and disorders of cerebral circulation, a person should receive 2-6 g of essential polyunsaturated fatty acids per day. These acids are necessary for the synthesis of:

Стероїдів Steroids

Адреналіну Adrenaline

Простагладинів Prostagladinov

Вітамінів групи D D vitamins

Жовчних кислот Bile acids

87 / 180
Жінка 35-ти років розпочала голодування. Депо яких поживних речовин використовується у початковий період голодування і як при цьому змінюється дихальний коефіцієнт (ДК)? A 35-year-old woman started fasting. What nutrient depot is used in the initial period of fasting and how does the respiratory coefficient (DC) change?

Жири, ДК наближається до 0,72 Fats, DC approaching 0.72

Білки, ДК наближається до 0,7 Proteins, DC approaching 0.7

Жири, ДК наближається до 0,85 Fats, DC approaching 0.85

Білки, ДК наближається до 1 Proteins, DC approaching 1

Вуглеводи, ДК наближається до 1 Carbs, DC approaching 1

88 / 180
На мікропрепараті серця розрізняємо кардіоміоцити зірчастої форми з центрально розташованим ядром, розвиненими гранулярною ендоплазматичною сіткою, апаратом Гольджі та специфічними гранулами. З цими клітинами пов’язана така функція: On the heart microsection, we can distinguish stellate-shaped cardiomyocytes with a centrally located nucleus, a developed granular endoplasmic reticulum, a Golgi apparatus and specific granules. The following function is associated with these cells:

Регенераторна Regenerative

Захисна Protective

Проведення імпульсу Pulse conduction

Скорочення Abbreviation

Ендокринна Endocrine

89 / 180
43-річний хворий надійшов в нефрологічне відділення з масивними набряками. Два роки лікувався амбулаторно, при цьому постійно відзначався підвищений артеріальний тиск. Двічі лікувався преднізолоном, з позитивним ефектом. У сечі: відносна щільність - 1017, білок - 4,0 г/л, еритроцити - 15-18 в полі зору (вилужені), лейкоцити - 5-7 в полі зору. Яка переважно функція нирок порушена у хворого? A 43-year-old patient was admitted to the nephrology department with massive edema. He was treated on an outpatient basis for two years, while elevated blood pressure was constantly noted. He was treated twice with prednisolone, with a positive effect. In the urine: relative density - 1017, protein - 4.0 g/l, erythrocytes - 15-18 in the field of vision (leachated), leukocytes - 5-7 in the field of vision. Which kidney function is mainly impaired in the patient?

Фільтраційна Filtering

Секреторна Secretary

Концентраційна Concentration

Реабсорбційна Reabsorption

Інкреторна Incretory

90 / 180
Під час трьох вагітностей у жінки спостерігалися викидні. З анамнезу відомо, що жінка протягом тривалого часу проживала в сім’ї, де була кішка. Яким одноклітинним паразитом, що міг бути причиною викиднів, могла заразитися жінка? During three pregnancies, the woman had miscarriages. It is known from the anamnesis that the woman lived for a long time in a family where there was a cat. What single-celled parasite that could be the cause of miscarriages, could the woman be infected?

Балантидій Balantidius

Трихомонада Trichomonad

Токсоплазма Toxoplasma

Лямблія Lamblia

Амеба Amoeba

91 / 180
Хворий звернувся до лікаря зі скаргами на пронос і біль в животі протягом 5-ти днів, підвищену температуру тіла до 37,5°С. Бактеріологічно встановлений діагноз амебна дизентерія. Вкажіть препарат вибору для лікування цього захворювання: The patient went to the doctor with complaints of diarrhea and abdominal pain for 5 days, elevated body temperature up to 37.5°C. Bacteriologically diagnosed amoebic dysentery Specify the drug of choice for the treatment of this disease:

Фурацилін Furacilin

Хітамін Chitamine

Ітраконазол Itraconazole

Ацикловір Acyclovir

Метронідазол Metronidazole

92 / 180
Хворій 39-ти років, яка протягом 8-ми років не може завагітніти, порадили звернутися до ендокринолога. При обстеженні у хворої виявлено екзофтальм, тремор повік, тахікардію. Захворювання якої ендокринної залози супроводжується такими симптомами? A 39-year-old patient who has not been able to get pregnant for 8 years was advised to consult an endocrinologist. During the examination, exophthalmos, tremor of the eyelids, and tachycardia were found in the patient. Which endocrine gland disease is accompanied by such symptoms?

Надниркових Adrenal

Епіфіза Pinephysis

Підшлункової Pancreatic

Статевих Sex

Щитоподібної Thyroid

93 / 180
На заняттях з лікувальної фізкультури лікар-фізіотерапевт запропонував юнакам відхилитися назад і дістати долонями до підлоги. Яка зв’язка запобігає надмірному розгинанню хребтового стовпа? During a physical therapy class, a physiotherapist suggested the young men lean back and touch the floor with their palms. What ligament prevents excessive extension of the spinal column?

Задня поздовжня Back Longitudinal

Жовта Yellow

Передня поздовжня Front longitudinal

Надостьова Nadostova

Міжпоперечна Intertransverse

94 / 180
Дитина 3-х років померла від гострої пневмонії на тлі хронічної серцевої недостатності. На аутопсії: дефект міжшлуночкової перегородки, стеноз устя легеневої артерії, гіпертрофія правого шлуночка серця, декстрапозиція аорти. Яка вада серця у дитини була встановлена в ході аутопсії? A 3-year-old child died of acute pneumonia on the background of chronic heart failure. At autopsy: defect of the interventricular septum, stenosis of the mouth of the pulmonary artery, hypertrophy of the right ventricle of the heart, dextraposition aorta. What heart defect was established in the child during the autopsy?

Синдром Марфана Marfan syndrome

Тетрада Фалло Tetrad of Fallot

Пентада Фалло Pentade Fallo

- -

Синдром Патау Patau syndrome

95 / 180
У хворого, який помер від уремії, на розтині виявлена деформація хребетного стовпа з різким обмеженням рухливості. Суглобові хрящі дрібних суглобів хребта зруйновані, є виражені ознаки тривалого поточного хронічного запалення в тканинах суглобів, порожнини суглобів заповнені сполучною тканиною, місцями кістковою з формуванням анкілозів. В аорті, серці та легенях наявні хронічне запалення та вогнищевий склероз. У нирках спостерігається амілоїдоз. Який діагноз у цьому випадку найімовірніший? In a patient who died of uremia, the autopsy revealed a deformation of the spinal column with a sharp limitation of mobility. The articular cartilages of the small joints of the spine are destroyed, there are pronounced signs of long-term ongoing chronic inflammation in the tissues of the joints, the joint cavities are filled with connective tissue, sometimes bone with the formation of ankylosis. Chronic inflammation and focal sclerosis are present in the aorta, heart, and lungs. Amyloidosis is observed in the kidneys. What is the most likely diagnosis in this case?

Ревматоїдний артрит Rheumatoid arthritis

Остеопетроз (мармурова хвороба) Osteopetrosis (marble disease)

Хвороба Педжета (деформівний остоз) Paget's disease (deformative ostosis)

Анкілозивний спондилоартрит (хвороба Бехтєрєва) Ankylosing spondylitis (Bekhterev's disease)

Паратиреоїдна остеодистрофія Parathyroid osteodystrophy

96 / 180
При дефіциті біотину синтез вищих жирних кислот знижується. Внаслідок недостатньої активності якого ферменту це відбувається? With a biotin deficiency, the synthesis of higher fatty acids decreases. This is due to insufficient activity of which enzyme?

Піруватдегідрогенази Pyruvate dehydrogenases

Ацетил-КоА-карбоксилази Acetyl CoA carboxylases

Цитратсинтетази Citrate synthetases

Еноїлредуктази Enoyl reductases

Бета-кетоацилредуктази Beta-ketoacyl reductases

97 / 180
Хворому поставили попередній діагноз інфаркт міокарда. Характерною ознакою цього захворювання є суттєве підвищення в крові активності: The patient was given a preliminary diagnosis of myocardial infarction. A characteristic feature of this disease is a significant increase in blood activity:

Г-6-ФДГ G-6-FDG

Альфа-амілази Alpha amylases

Каталази Catalase

Кретинфосфокінази Cretin phosphokinase

Аргінази Arginase

98 / 180
Хворому на крупозну пневмонію внутрішньом’язово ввели бензилпеніцилін-натрій. Через кілька хвилин у пацієнта розвинувся анафілактичний шок. Який лікарський засіб необхідно ввести хворому? Benzylpenicillin sodium was administered intramuscularly to a patient with croup pneumonia. After a few minutes, the patient developed an anaphylactic shock. What drug should be administered to the patient?

Кофеїн-бензоат натрію Caffeine sodium benzoate

Мезатон Mesaton

Адреналіну гідрохлорид Adrenaline hydrochloride

Норадреналіну гідротартрат Noradrenaline hydrotartrate

Ефедрин Ephedrine

99 / 180
Хворий переніс повторний інтрамуральний інфаркт міокарда. Після лікування та реабілітації був виписаний у задовільному стані під нагляд дільничного терапевта. Через 2 роки загинув у автомобільній катастрофі. Який характер патологічного процесу в міокарді було встановлено на розтині? The patient suffered a repeated intramural myocardial infarction. After treatment and rehabilitation, he was discharged in a satisfactory condition under the supervision of a district therapist. He died in a car accident 2 years later. What is the nature of the pathological process was established in the myocardium at autopsy?

Атрофія Atrophy

Дрібновогнищевий кардіосклероз Microfocal cardiosclerosis

Великовогнищевий кардіосклероз Multifocal cardiosclerosis

Гіперплазія Hyperplasia

Некроз Necrosis

100 / 180
У хворого діагностовано пухлину мозку, яка розміщена в ділянці 'пташиної шпори'. Порушення якої функції виникне у хворого, якщо пухлина буде активно розвиватися? The patient has been diagnosed with a brain tumor, which is located in the area of the 'bird's spur'. Which function will be impaired in the patient if the tumor is actively developing?

Зір Sight

Нюх Sniff

Смак Taste

Дотикова чутливість Touch sensitivity

Слух Hearing

101 / 180
У хворого з хронічним гіперацидним гастритом з’явився біль у суглобах. Для полегшення болю, враховуючи супутню патологію, був призначений целекоксиб. Вибіркова дія цього препарату на певний фермент забезпечує відсутність впливу на слизову шлунка. Назвіть цей фермент: У хворого з хронічним гіперацидним гастритом з’явився біль у суглобах. Для полегшення болю, враховуючи супутню патологію, був призначений целекоксиб. Вибіркова дія цього препарату на певний фермент забезпечує відсутність впливу на слизову шлунка. Назвіть цей фермент:

Калікреїн Калікреїн

Фосфоліпаза С Фосфоліпаза С

Фосфоліпаза А2 Фосфоліпаза А2

Циклооксигеназа 2 Циклооксигеназа 2

Циклооксигеназа 1 Циклооксигеназа 1

102 / 180
Чим пояснити той факт, що для лікування туберкульозу доза ізоніазиду підбирається індивідуально, з обов’язковим контролем після перших прийомів препарату його вмісту в сечі? How to explain the fact that for the treatment of tuberculosis, the dose of isoniazid is selected individually, with mandatory monitoring of its content in the urine after the first doses of the drug?

Подразнювальною дією препарату The irritating effect of the drug

Розвитком ниркової недостатності Development of kidney failure

Гіперглікемією, що виникає на тлі приймання препарату Hyperglycemia occurring against the background of taking the drug

Генетично обумовленою швидкістю ацетилювання препарату у різних людей Genetically determined rate of drug acetylation in different people

Розвитком гемолітичної анемії Development of hemolytic anemia

103 / 180
У хворого щорічно навесні та на початку літа в період цвітіння трав і дерев розвивається гостре катаральне запалення кон’юнктиви очей та слизової носової порожнини. Активація та екзоцитоз яких клітинних елементів лежить в основі цього синдрому? The patient develops acute catarrhal inflammation of the conjunctiva of the eyes and mucous membrane of the nasal cavity every year in spring and early summer during the flowering period of grasses and trees. Activation and exocytosis of which cellular elements underlies this syndrome?

Нейтрофілів Neutrophils

Тромбоцитів Platelets

Макрофагів Macrophages

Ендотеліальних клітин Endothelial cells

Тканинних базофілів Tissue basophils

104 / 180
Хворий 13-ти років скаржиться на загальну слабкість, запаморочення, втомлюваність. Спостерігається відставання у розумовому розвитку. При обстеженні виявлено високу концентрацію валіну, ізолейцину, лейцину в крові та сечі. Сеча має специфічний запах. Що може бути причиною такого стану? A 13-year-old patient complains of general weakness, dizziness, fatigue. There is a delay in mental development. The examination revealed a high concentration of valine, isoleucine, leucine in the blood and urine. Urine has a specific smell. What could be the cause of this condition?

Гістидинемія Histidinemia

Хвороба кленового сиропу Maple syrup disease

Базедова хвороба Bazed disease

Хвороба Аддісона Addison's disease

Тирозиноз Tyrosinosis

105 / 180
У дитини 6-ти років захворювання почалося гостро з різкого катару в зіві та на мигдаликах, який поширився на слизову оболонку рота, язик ('малиновий язик') та глотку. На поверхні мигдаликів некрози. Місцями внаслідок відторгнення некротичних мас утворюються виразки. Шийні лімфовузли збільшені. На тілі, за винятком носогубного трикутника, спостерігається дрібнокрапковий висип яскраво-червоного кольору. Яке захворювання можна припустити? In a 6-year-old child, the disease began acutely with a sharp catarrh in the throat and tonsils, which spread to the mucous membrane of the mouth, tongue ('raspberry tongue') and pharynx. Necrosis on the surface of the tonsils. Ulcers form in places as a result of the rejection of necrotic masses. Cervical lymph nodes are enlarged. On the body, except for the nasolabial triangle, there is a small-dot rash of bright red color. What disease can be assumed?

Ангіна Angina

Скарлатина Scarlatina

Дифтерія Diphtheria

Кір Measles

Менінгококовий назофарингіт Meningococcal nasopharyngitis

106 / 180
Клінічне обстеження хворого дозволило встановити попередній діагноз рак шлунка. В шлунковому соці виявлено молочну кислоту. Який тип катаболізму глюкози має місце у ракових клітинах? Клінічне обстеження хворого дозволило встановити попередній діагноз рак шлунка. В шлунковому соці виявлено молочну кислоту. Який тип катаболізму глюкози має місце у ракових клітинах?

Глюкозо-аланіновий цикл Глюкозо-аланіновий цикл

Анаеробний гліколіз Анаеробний гліколіз

Аеробний гліколіз Аеробний гліколіз

Глюконеогенез Глюконеогенез

Пентозофосфатний цикл Пентозофосфатний цикл

107 / 180
При вимірюванні артеріального тиску у чоловіка 56-ти років встановлено зростання діастолічного артеріального тиску до 100 мм рт.ст. Від якого з наведених факторів переважно залежить величина діастолічного артеріального тиску? При вимірюванні артеріального тиску у чоловіка 56-ти років встановлено зростання діастолічного артеріального тиску до 100 мм рт.ст. Від якого з наведених факторів переважно залежить величина діастолічного артеріального тиску?

Величини кінцево-діастолічного об’єму лівого шлуночка Values of the end-diastolic volume of the left ventricle

Периферичного опору судин Peripheral vascular resistance

Величини ударного об’єму лівого шлуночка Left ventricular stroke volume values

Швидкості кровотоку Blood flow rates

Об’єму циркулюючої крові Circulating blood volume

108 / 180
Артеріальна гіпертензія у хворої 44-х років обумовлена наявністю феохромоцитоми - пухлини мозкового шару наднирників. Антигіпертензивні засоби якої групи найдоцільніше буде призначити? Hypertension in a 44-year-old patient is due to the presence of a pheochromocytoma - a tumor of the medulla of the adrenal glands. Which group of antihypertensive drugs would be the most appropriate to prescribe?

Симпатолітики Sympatholytics

Антагоністи кальцію Calcium antagonists

Альфа-адреноблокатори Alpha-adrenoblockers

Гангліоблокатори Ganglioblockers

Бета-адреноблокатори Beta blockers

109 / 180
Бактеріологічний метод діагностики був використаний для підтвердження діагнозу газова гангрена у хворого. Які живильні середовища необхідно використовувати для культивування збудника в цьому випадку? The bacteriological method of diagnosis was used to confirm the diagnosis of gas gangrene in the patient. What nutrient media should be used for the cultivation of the pathogen in this case?

Ендо, Левіна, Плоскірєва Endo, Levina, Ploskireva

Лужний агар Alkaline Agar

ЖСА, кров’яний агар ZHA, blood agar

МПА, МПБ MPA, MPB

Вільсона-Блера, Кітта-Тароцці Wilson-Blair, Kitt-Tarozzi

110 / 180
Хворому з діагнозом цукровий діабет II типу ендокринолог призначив глібенкламід. Вкажіть основний механізм дії цього засобу: Хворому з діагнозом цукровий діабет II типу ендокринолог призначив глібенкламід. Вкажіть основний механізм дії цього засобу:

Підсилює захоплення глюкози периферичними тканинами Підсилює захоплення глюкози периферичними тканинами

Стимулює секрецію інсуліну бета- клітинами острівців Лангерганса Стимулює секрецію інсуліну бета- клітинами острівців Лангерганса

Пригнічує глюконеогенез Пригнічує глюконеогенез

Підсилює метаболізм глюкози Підсилює метаболізм глюкози

Активує транспорт глюкози в клітину Активує транспорт глюкози в клітину

111 / 180
Хворому 35-ти років для обстеження очного дна був призначений атропіну сульфат у вигляді очних крапель. Для відновлення акомодації йому закрапали пілокарпіну гідрохлорид, але це не дало бажаного ефекту. Що є причиною відсутності ефекту? Хворому 35-ти років для обстеження очного дна був призначений атропіну сульфат у вигляді очних крапель. Для відновлення акомодації йому закрапали пілокарпіну гідрохлорид, але це не дало бажаного ефекту. Що є причиною відсутності ефекту?

Тахіфілаксія Тахіфілаксія

Двосторонній антагонізм Двосторонній антагонізм

Односторонній антагонізм Односторонній антагонізм

Звикання Звикання

Синергізм Синергізм

112 / 180
До хірургічного відділення ЦРЛ надійшов хворий з колотою раною стопи, яку він отримав під час косовиці. Який специфічний препарат необхідно застосувати з метою екстреної пасивної імунопрофілактики правця? A patient was admitted to the surgical department of the Central Hospital with a puncture wound of the foot, which he received during a bloodbath. What specific drug should be used for the purpose of emergency passive immunoprophylaxis of tetanus?

Антибіотики Antibiotics

Вакцина АКДП AKDP vaccine

Анатоксин Anatoxin

Антитоксична сироватка Antitoxic Serum

Протиправцева вакцина Tetanus vaccine

113 / 180
В медико-генетичну консультацію за рекомендацією андролога звернувся чоловік 35-ти років з приводу відхилень фізичного і психічного розвитку. Об’єктивно встановлено: високий зріст, астенічна будова тіла, гінекомастія, розумова відсталість. При мікроскопії клітин слизової оболонки ротової порожнини знайдено в 30% статевий хроматин (одне тільце Барра). Який діагноз найімовірніший? On the recommendation of an andrologist, a 35-year-old man applied for medical and genetic counseling due to abnormalities in physical and mental development. Objectively established: tall height, asthenic body structure , gynecomastia, mental retardation. Microscopy of the cells of the mucous membrane of the oral cavity revealed 30% sex chromatin (one Barr body). What is the most likely diagnosis?

Хвороба Дауна Down's disease

Синдром Клайнфельтера Klinefelter syndrome

Хвороба Реклінгаузена Recklinghausen's disease

Синдром Ді Джорджі Di Giorgi Syndrome

Хвороба Іценка-Кушинга Itsenko-Cushing disease

114 / 180
В психіатрії для лікування ряду захворювань ЦНС використовують біогенні аміни. Вкажіть препарат цієї групи, який є медіатором гальмування: Biogenic amines are used in psychiatry to treat a number of CNS diseases. Specify the drug of this group that is a mediator of inhibition:

Гамма-аміномасляна кислота Gamma-aminobutyric acid

Серотонін Serotonin

Гістамін Histamine

Дофамін Dopamine

Таурин Taurine

115 / 180
В експерименті на жабі вивчали міотатичний рефлекс. Однак при роз-тягненні скелетного м’яза, його рефлекторне скорочення не відбулося. На по-рушення функції яких рецепторів слід звернути увагу? In an experiment on a frog, the myotatic reflex was studied. However, when the skeletal muscle was stretched, its reflex contraction did not occur. The dysfunction of which receptors should be considered attention?

Тактильних Tactile

Суглобових Articular

Больових Bolovy

Сухожильних органів Гольджі Golgi tendon organs

М’язових веретен Muscle spindles

116 / 180
У жінки під час мейозу відбулося порушення розходження аутосом. Утворилася яйцеклітина із зайвою 18-ю хромосомою. Яйцеклітина запліднюється нормальним сперматозооном. У майбутньої дитини буде синдром: During meiosis, a woman had a violation of autosome segregation. An egg cell with an extra 18th chromosome was formed. The egg cell is fertilized by a normal spermatozoon. The future child will have the syndrome:

Патау Patau

Клайнфельтера Klinefelter

Дауна Down

Шерешевського-Тернера Shereshevsky-Turner

Едвардса Edwards

117 / 180
На розтині звертає на себе увагу наступне: тіло молодого чоловіка високого зросту, виражена блідість та еластичність шкіри, доліхоцефалічна будова голови, подовження і контрактура пальців верхніх і нижніх кінцівок (нагадують кінцівки павука), незначне вдавлення грудини всередину, помірно виражений сколіоз. При дослідженні органів грудної порожнини виявили розрив аневризми висхідної частини аорти. Яке захворювання, найімовірніше, мало місце в цьому випадку? At the autopsy, the following draws attention: the body of a tall young man, pronounced paleness and elasticity of the skin, dolichocephalic structure of the head, elongation and contracture of the fingers of the upper and lower limbs ( resemble the limbs of a spider), a slight depression of the sternum inward, moderately pronounced scoliosis. An examination of the chest cavity revealed a rupture of an aneurysm of the ascending part of the aorta. What disease most likely occurred in this case?

Атеросклероз аорти Aortic atherosclerosis

Синдром Гудпасчера Goodpasture syndrome

Сифілітичний мезоаортит Syphilitic mesoaortitis

- -

Синдром Марфана Marfan syndrome

118 / 180
Пацієнт був доставлений до лікарні з такими симптомами: запаморочення, сухість в роті, зіниці сильно розширені, порушення акомодації, тахікардія, утруднення сечовипускання, атонія кишечника. Передозування якого препарату могло викликати ці симптоми? The patient was brought to the hospital with the following symptoms: dizziness, dry mouth, greatly dilated pupils, impaired accommodation, tachycardia, difficulty urinating, intestinal atony. Overdose of which drug could have caused these symptoms?

Празозин Prazozin

Фуросемід Furosemide

Атропіну сульфат Atropine sulfate

Клофелін Clofelin

Каптоприл Captopril

119 / 180
На розтині тіла померлого виявлено, що вся права легеня збільшена, щільна, на плеврі нашарування фібрину, на розрізі тканина сірого кольору, з якої стікає каламутна рідина. Для якого захворювання легенів характерна така картина? At the autopsy of the dead body, it was found that the entire right lung was enlarged, dense, on the pleura layers of fibrin, on the cross section, the tissue was gray in color, from which a cloudy liquid was flowing. For which lung disease is characterized by such a picture?

Крупозна пневмонія Croup pneumonia

Гангрена легені Gangrene of the lungs

Фіброзивний альвеоліт Fibrosing alveolitis

Інтерстиціальна пневмонія Interstitial pneumonia

Вогнищева пневмонія Focal pneumonia

120 / 180
При деяких гельмінтозах людина може сама виявити гельмінта, оскільки зрілі членики збудника можуть активно виповзати з ануса людини. Це характерно для: With some helminthiasis, a person can detect the helminth himself, as mature members of the pathogen can actively crawl out of the anus of a person. This is characteristic of:

Ехінококозу Echinococcosis

Теніозу Taeniosis

Дифілоботріозу Diphyllobotriosis

Гіменолепідозу Hymenolepidosis

Теніаринхозу Taeniarhynchosis

121 / 180
Чоловік 35-ти років захворів гостро, відзначалося підвищення температури до 39°С, з’явилися нежить, кашель, сльозотеча. При огляді слизова носоглотки набрякла, гіперемована з рясним слизовиділенням. Який вид запалення розвинувся в носоглотці? A 35-year-old man became acutely ill, the temperature rose to 39°C, a runny nose, cough, and lacrimation appeared. On examination, the nasopharyngeal mucosa was swollen, hyperemic with abundant mucus secretion. What kind of inflammation developed in the nasopharynx?

Геморагічне Hemorrhagic

Фібринозне Fibrinous

Гнійне Purulent

Серозне Serious

Катаральне Catarrhal

122 / 180
У чоловіка 33-х років як наслідок спинномозкової травми порушена больова та температурна чутливість, що обумовлено пошкодженням такого шляху: A 33-year-old man has impaired pain and temperature sensitivity as a result of a spinal cord injury, which is caused by damage to the following pathway:

Заднього спіномозочкового Posterior spinocerebellum

Медіального спінокортикального Medial spinocortical

Латерального спінокортикального Lateral spinocortical

Переднього спіномозочкового Anterior spinocerebellum

Спіноталамічного Spinothalamic

123 / 180
Під час бігу на короткі дистанції у нетренованої людини виникає м’язова гіпоксія. До накопичення якого метаболіту в м’язах це призводить? During short-distance running in an untrained person, muscle hypoxia occurs. What metabolite does this lead to the accumulation of in the muscles?

Кетонових тіл Кетонових тіл

Ацетил-КоА Ацетил-КоА

Глюкозо-6-фосфату Глюкозо-6-фосфату

Оксалоацетату Оксалоацетату

Лактату Лактату

124 / 180
Студент перкуторно визначає межу серця, яка проектується на передню грудну клітку на рівні хрящів третього ребра. Яку межу серця визначив студент? Студент перкуторно визначає межу серця, яка проектується на передню грудну клітку на рівні хрящів третього ребра. Яку межу серця визначив студент?

Ліву Ліву

Нижню Нижню

Праву Праву

Верхню Верхню

Верхівку Верхівку

125 / 180
У хворого з підозрою на «озену» з носоглотки були виділені грамнегативні палички, які утворювали капсулу на поживному середовищі. Які мікроорганізми спричинили хворобу? Gram-negative bacilli were isolated from the nasopharynx of a patient suspected of having ozena, which formed a capsule on a nutrient medium. What microorganisms caused the disease?

Шигели Schigels

Хламідії Chlamydia

Сальмонели Salmonella

Мікоплазми Mycoplasma

Клебсієли Klebsiels

126 / 180
Жінка літнього віку перенесла сильний стрес. У крові різко збільшилась концентрація адреналіну і норадреналіну. Які ферменти каталізують процес інактивації катехоламінів? An elderly woman suffered severe stress. The concentration of adrenaline and noradrenaline in the blood increased sharply. What enzymes catalyze the process of catecholamine inactivation?

Пептидази Peptidases

Тирозиназа Tyrosinase

Карбоксилази Карбоксилази

Моноамінооксидази Моноамінооксидази

Глікозидази Глікозидази

127 / 180
У пацієнта, який тривалий час вживає препарати, що блокують вироблення ангіотензину II, виникли брадикардія, порушення серцевого ритму. Можливою причиною цих розладів є: У пацієнта, який тривалий час вживає препарати, що блокують вироблення ангіотензину II, виникли брадикардія, порушення серцевого ритму. Можливою причиною цих розладів є:

Гіперкальціємія Гіперкальціємія

Гіперкаліємія Гіперкаліємія

Гіпокальціємія Гіпокальціємія

Гіпокаліємія Гіпокаліємія

Гіпернатріємія Гіпернатріємія

128 / 180
Хворому 68-ми років в комплекс лікування атеросклерозу, ускладненого ішемічною хворобою серця, лікар включив гіполіпідемічний засіб, який знижує вміст в крові переважно тригліцеридів. Який із вказаних препаратів було призначено хворому? To a 68-year-old patient, in the complex treatment of atherosclerosis complicated by coronary heart disease, the doctor included a hypolipidemic drug that reduces the content of triglycerides in the blood. Which of the specified drugs was prescribed for the patient?

Преднізолон Prednisone

Анаприлін Anaprilin

Глібенкламід Glibenclamide

Інсулін Insulin

Фенофібрат Fenofibrate

129 / 180
У дитини, яка часто хворіє на ангіни та фарингіти, відзначається збіль-шення лімфовузлів і селезінки. Зовнішній вигляд характеризується пастозністю та блідістю, м’язова тканина розвинена слабко. У крові спостерігається лімфоцитоз. Як називається такий вид діатезу? A child who often suffers from sore throats and pharyngitis has an enlarged lymph nodes and spleen. The appearance is characterized by pastiness and pallor, the muscle tissue is poorly developed. Lymphocytosis is observed in the blood. What is the name of this type of diathesis?

Нервово-артритичний Nervous-arthritic

Ексудативно-катаральний Exudative-catarrhal

Геморагічний Hemorrhagic

Астенічний Asthenic

Лімфатико-гіпопластичний Lymphatic-hypoplastic

130 / 180
У здорової дорослої людини проводять зондування порожнин серця і великих судин. Де розташований зонд, якщо протягом серцевого циклу зареєстровані зміни тиску від 0 до 120 мм рт.ст.? In a healthy adult, sounding of the heart cavity and large vessels is performed. Where is the probe located if pressure changes from 0 to 120 mm Hg are registered during the cardiac cycle?

Передсердя Atrial

Легенева артерія Pulmonary artery

Аорта Aorta

Лівий шлуночок Left Ventricle

Правий шлуночок Right wing night

131 / 180
Під час гінекологічного обстеження пацієнтці був поставлений діагноз ендометрит (запалення ендометрію). Яка частина маткової стінки уражена запальним процесом? During a gynecological examination, the patient was diagnosed with endometritis (inflammation of the endometrium). Which part of the uterine wall is affected by the inflammatory process?

Навколоматкова клітковина Uterine fiber

Серозна оболонка Serous membrane

Слизова оболонка Mucosa

Адвентиційна оболонка Adventitous membrane

М ’язова оболонка Muscle

132 / 180
Хворому на туберкульоз призначено бактерицидний антибіотик, побічним ефектом якого є забарвлення біологічних рідин у червоний колір. Визначте препарат: A tuberculosis patient is prescribed a bactericidal antibiotic, the side effect of which is a red coloring of biological fluids. Identify the drug:

Протіонамід Prothionamide

Рифампіцин Rifampicin

Стрептоміцину сульфат Streptomycin sulfate

Етамбутол Etambutol

Ізоніазид Isoniazid

133 / 180
Аміак є дуже отруйною речовиною, особливо для нервової системи. Яка речовина бере особливо активну участь у знешкодженні аміаку в тканинах мозку? Ammonia is a very poisonous substance, especially for the nervous system. What substance is particularly active in neutralizing ammonia in brain tissue?

Пролін Proline

Аланін Alanine

Лізин Lysine

Глутамінова кислота Glutamic acid

Гістидин Histidine

134 / 180
Офтальмолог з діагностичною метою (розширення зіниць для огляду очного дна) використав 1% розчин мезатону. Мідріаз, викликаний препаратом, обумовлений: The ophthalmologist used a 1% mesaton solution for diagnostic purposes (pupil dilation to examine the fundus). Mydriasis caused by the drug is due to:

Активацією α₁-адренорецепторів Activation of α₁-adrenoceptors

Активацією α₂-адренорецепторів Activation of α₂-adrenoceptors

Активацією М-холінорецепторів Activation of M-cholinergic receptors

Активацією β₁ -адренорецепторів By activation of β₁ -adrenoceptors

Блокадою α₁-адренорецепторів By blocking α₁-adrenergic receptors

135 / 180
В результаті травми у чоловіка 47- ми років пошкоджені передні корінці спинного мозку. Відростки яких нейронів пошкоджені? As a result of an injury, a 47-year-old man has damaged the anterior roots of the spinal cord. Which neuron processes are damaged?

Дендрити й аксони чутливих псевдоуніполярних нейронів Dendrites and axons of sensitive pseudounipolar neurons

Дендрити чутливих псевдоуніполярних нейронів Dendrites of sensitive pseudounipolar neurons

Дендрити рухових нейронів й аксони ядер бокових стовпів Dendrites of motor neurons and axons of nuclei of lateral columns

Аксони чутливих псевдоуніполярних нейронів Axons of sensitive pseudounipolar neurons

Аксони нейронів рухових соматичних та вегетативних ядер Axons of neurons of motor somatic and vegetative nuclei

136 / 180
У дитини 7-ми років підвищилася температура тіла до 39°С, з’явилися нежить, кон’юнктивіт і кашель. На шкірі відзначалися великоплямисті висипи, при огляді порожнини рота спостерігаються білуваті висипи на слизовій оболонці щік. Раптово з’явилося важке дихання і настала смерть при явищах асфіксії. Який діагноз найімовірніший? A 7-year-old child's body temperature rose to 39°C, a runny nose, conjunctivitis, and cough appeared. Large spotted rashes were noted on the skin, during examination in the oral cavity, whitish rashes are observed on the mucous membrane of the cheeks. Heavy breathing suddenly appeared and death occurred due to asphyxiation. What is the most likely diagnosis?

Скарлатина Scarlatina

Кір Measles

Менінгококовий назофарингіт Meningococcal nasopharyngitis

Дифтерія Diphtheria

Грип Flu

137 / 180
У відділення реанімації надійшов хворий після ДТП з однобічним пневмотораксом. Який вид дихання спостерігається у цьому випадку? A patient was admitted to the intensive care unit after a road accident with unilateral pneumothorax. What type of breathing is observed in this case?

Глибоке часте Deep Frequent

Поверхневе Superficial

Поверхневе рідке Surface liquid

Асфіктичне Asphyxic

Поверхневе часте Surface frequent

138 / 180
Під дією УФ-опромінення та інших факторів можуть відбуватися зміни в структурі ДНК. Репарація молекули ДНК досягається узгодженою дією всіх наступних ферментів, ЗА ВИНЯТКОМ: Under the influence of UV-irradiation and other factors, changes in the structure of DNA can occur. Repair of the DNA molecule is achieved by the coordinated action of all the following enzymes, EXCEPT:

Аміноацил-тРНК-синтетази Aminoacyl-tRNA synthetases

ДНК-глікозидази DNA glycosidases

Ендонуклеази Endonucleases

ДНК-лігази DNA ligases

ДНК-полімерази DNA polymerases

139 / 180
Реалізація загального адаптаційного синдрому здійснюється переважно через нейроендокринну систему. Якій з ланок цієї системи належить провідна роль у патогенезі реакції, що розвивається? The implementation of the general adaptation syndrome is carried out mainly through the neuroendocrine system. Which of the links of this system has the leading role in the pathogenesis of the developing reaction?

Гіпофізарно-адреногенітальна Pituitary-adrenogenital

Гіпофізарно-тиреоїдна Pituitary-thyroid

Гіпофізарно-адреналова Pituitary-adrenal

Гіпофізарно-інсулярна Pituitary-insular

Гіпофізарно-юкстагломерулярна Pituitary-juxtaglomerular

140 / 180
Кросинговер - це обмін ділянками гомологічних хромосом у процесі клітинного поділу, переважно в профазі першого мейотичного поділу, іноді в мітозі. Від чого залежить частота кросинговеру? Crossing over is the exchange of sections of homologous chromosomes in the process of cell division, mainly in the prophase of the first meiotic division, sometimes in mitosis. What does the frequency of crossing over depend on?

Від зовнішніх факторів From external factors

Від довжини хромосоми From chromosome length

Від кількості хромосом From the number of chromosomes

Від відстані між генами From the distance between genes

Від типу хромосоми From chromosome type

141 / 180
У людини вимірюють енерговитрати натщесерце, лежачи, в умовах фізичного і психічного спокою, при температурі комфорту. В який час енерговитрати будуть найбільшими? A person's energy expenditure is measured on an empty stomach, lying down, in conditions of physical and mental rest, at a comfortable temperature. At what time will the energy expenditure be the greatest?

17-18 годин вечора 17-18 hours in the evening

3-4 години ранку 3-4 am

10-12 годин дня 10-12 hours of the day

14-16 годин дня 14-16 hours of the day

7-8 годин ранку 7-8 am

142 / 180
Відомо, що в ході альтерації у вогнищі запалення утворюються біологічно активні речовини-медіатори запалення, які відіграють важливу роль у патогенезі цього патологічного процесу. До утворення яких медіаторів призводить активація ліпооксигенази? It is known that in the course of alteration in the focus of inflammation, biologically active substances-mediators of inflammation are formed, which play an important role in the pathogenesis of this pathological process. The formation of which mediators leads to activation lipoxygenase?

Простагландинів Prostaglandins

Гістаміну Histamine

Лейкотрієнів Leukotrienes

Простацикліну Prostacyclin

Тромбоксану Tromboxane

143 / 180
Внаслідок захворювання нирок у пацієнта відзначаються набряки. В аналізах сечі визначається масивна протеїнурія. Який механізм є основним у виникненні набряків у такого пацієнта? As a result of kidney disease, the patient has edema. Massive proteinuria is detected in urine tests. What mechanism is the main cause of edema in such a patient?

Зниження онкотичного тиску тканин Reduction of tissue oncotic pressure

Зниження фільтраційного тиску в нирках Decreased filtration pressure in the kidneys

Підвищення осмотичного тиску плазми крові Increased blood plasma osmotic pressure

Зниження онкотичного тиску плазми крові Reduction of oncotic pressure of blood plasma

Зниження онкотичного тиску лімфи Decreased lymph oncotic pressure

144 / 180
У дитини при обстеженні виявлено набряклість обличчя та розширення вен верхньої частини тулуба. На рентгенограмі спостерігається збільшення тіні органів середостіння. Ураження якого органу імуногенезу призвело до виникнення клінічних проявів захворювання? During the child's examination, swelling of the face and dilatation of the veins of the upper part of the body were detected. An increase in the shadow of the mediastinal organs is observed on the X-ray. Damage to which organ of immunogenesis led to the emergence of clinical manifestations of the disease?'

Діафрагмальні лімфатичні вузли Diaphragmatic lymph nodes

Кістковий мозок грудини Bone marrow of sternum

Тимус Thymus

Задні середостінні лімфатичні вузли Posterior mediastinal lymph nodes

Передні середостінні лімфатичні вузли Anterior mediastinal lymph nodes

145 / 180
Тривалий вплив на організм людини токсичних речовин призвів до руйнування органел, які відповідають за синтез білків у гепатоцитах печінки. Які органели здійснюють синтез білків у гепатоцитах? Long-term exposure to toxic substances on the human body led to the destruction of organelles responsible for protein synthesis in hepatocytes of the liver. What organelles carry out protein synthesis in hepatocytes?

Мітохондрії Mitochondria

Лізосоми Lysosomes

Агранулярна ендоплазматична сітка Agranular endoplasmic reticulum

Пероксисоми Peroxisomes

Рибосоми Ribosomes

146 / 180
У хворого після травми в скроневій ділянці виявлена епідуральна гематома. Яка артерія була пошкоджена? The patient had an epidural hematoma after an injury in the temporal area. Which artery was damaged?

Нижня барабанна артерія Inferior tympanic artery

Верхня барабанна артерія Superior tympanic artery

Глибока скронева артерія Deep temporal artery

Глибока вушна артерія Deep auricular artery

Середня оболонна артерія Medium meningeal artery

147 / 180
У хворого після травми втрачена чутливість шкіри на долонній поверхні 5-го пальця лівої кисті. Який нерв пошкоджений? After an injury, the patient has lost skin sensitivity on the palm surface of the 5th finger of the left hand. Which nerve is damaged?

N. radialis N. radialis

N. ахillaris N. ahillaris

N. ulnaris N. ulnaris

N. musculo-cutaneus N. musculo-cutaneus

N. теdianus N. tedianus

148 / 180
Для прискорення загоєння рани слизової оболонки в ротовій порожнині хворому призначено препарат, який являє собою термостабільний білок, що міститься у людини в сльозах, слині, грудному молоці матері, а також його можна виявити в свіжознесеному курячому яйці. Відомо, що він є фактором природної резистентності організму. Назвіть цей білок: To accelerate the healing of the wound of the mucous membrane in the oral cavity, the patient is prescribed a drug that is a heat-stable protein contained in human tears, saliva, mother's breast milk, and it can also be found in a freshly laid chicken egg. It is known to be a factor in the body's natural resistance. Name this protein:

Iнтерлейкін Interleukin

Комплемент Complement

Iнтерферон Interferon

Iманін Imanin

Лізоцим Lysozyme

149 / 180
В тубдиспансері у хворого на інфільтративну форму туберкульозу легень, який лікувався ізоніазидом, проявилися симптоми В6-гіповітамінозу. Чому ізоніазид призводить до цього явища? In the tube dispensary, a patient with an infiltrative form of pulmonary tuberculosis, who was treated with isoniazid, developed symptoms of B6 hypovitaminosis. Why does isoniazid lead to this phenomenon?

Iзоніазид є антагоністом вітаміну В6 Isoniazid is an antagonist of vitamin B6

Прискорюється біотрансформація Biotransformation is accelerating

Сповільнюється всмоктування вітаміну Vitamin absorption slows down

Утворюється міцний зв’язок з білками плазми крові A strong bond with blood plasma proteins is formed

Прискорюється елімінація Elimination is accelerating

150 / 180
В аналізі крові 35-річного хворого: Нb = 58 г/л, еритроцити = 1,3•1012/л, колірний показник = 1,3, лейкоцити = 2,8•109/л, тромбоцити = 1,1•109/л, ретикулоцити = 2%, ШОЕ = 35 мм/год. Визначаються полісегментовані нейтрофіли, а також тільця Жоллі, кільця Кебота, мегалоцити. Яка це анемія? In the blood analysis of a 35-year-old patient: Hb = 58 g/l, erythrocytes = 1.3•1012/l, color index = 1.3, leukocytes = 2.8•109/l, platelets = 1.1•109/l, reticulocytes = 2%, ESR = 35 mm/h. Polysegmented neutrophils are detected, as well as Jolly bodies, Cabot rings, and megalocytes. What kind of anemia is this?

Гемолітична Hemolytic

Гіпопластична Hypoplastic

В12-фолієводефіцитна B12-folate deficient

Постгеморагічна Posthemorrhagic

Залізодефіцитна Iron deficiency

151 / 180
72-year-old female patient suffers from a long-standing heart failure. She presents to the hospital with a cough and dyspnea on exertion for 1 week. Her symptoms worsen at night and she has noticed that her sputum is pink and frothy. Chest auscultation reveals bilateral line crepitations over the lung bases. The physician prescribes a drug that reduces preload. Which of the following is the most likely drug prescribed by the physician? 72-річна жінка страждає на тривалу серцеву недостатність. Вона звернулася до лікарні з кашлем і задишкою під час фізичного навантаження протягом 1 тижня. Симптоми посилюються вночі, вона помітила, що мокрота рожева та піниста Аускультація грудної клітки виявляє двосторонню крепітацію над основами легенів Лікар призначає препарат, що зменшує переднавантаження Що з перерахованого є найбільш імовірним препаратом, призначеним лікарем?

Furosemide Фуросемід

Clopamide Клопамід

Spironolactone Спіронолактон

Triamterene Тріамтерен

cetazolamide цетазоламід

152 / 180
A 12-year-old boy presents with progressive clumsiness and difficulty walking. He walks 'like a drunken-man' and has experienced frequent falls. His muscle tone and strength in all 4 limbs are slightly decreased. When he is asked to stand with his eyes closed and with both feet close together, he sways from side to side, unable to stand still. Which of the following brain regions is most likely affected and caused the symptoms described above? У 12-річного хлопчика спостерігається прогресуюча незграбність і труднощі при ходьбі. Він ходить 'як п'яний' і часто падає. Його м'язовий тонус і сила в усіх 4 кінцівках трохи знижується. Коли його просять стояти із закритими очима та зближеними ногами, він хитається з боку в бік, не в змозі стояти на місці. Яка з наступних областей мозку найбільш ймовірно уражена та викликав описані вище симптоми?

Cerebellum Мозочок

Reticular formation Ретикулярна формація

Red nucleus Червоне ядро

Substantia nigra Чорна субстанція

Right hemisphere Права півкуля

153 / 180
A doctor refers the patient to a gastroenterologist for a stomach acid test and an upper gastrointestinal endoscopy, which revealed that this patient is a heavy acid producer and has a gastric peptic ulcer. Which of the following is the most likely causative organism for this patient’s condition? Лікар направляє пацієнта до гастроентеролога для аналізу шлункової кислоти та ендоскопії верхніх відділів шлунково-кишкового тракту, які показали, що цей пацієнт є сильним кислотоутворювачем і має шлунковий пептична виразка. Що з перерахованого є найімовірнішим збудником цього стану пацієнта?

Salmonella Сальмонела

Shigella Шигелла

Leptospira Лептоспіра

Helicobacter Helicobacter

Listeria Лістерія

154 / 180
A 21-year-old woman who is a medical student is undergoing evaluation after sticking herself with a needle while drawing blood from a patient. In this case a medical professional is at high risk of getting a blood-transmitted infection. Which of the following diseases is least likely to be transmitted via blood? 21-річна жінка, яка є студенткою медичного факультету, проходить обстеження після того, як уколовся голкою під час забору крові у пацієнта. У цьому випадку медичний професіонал має високий ризик отримати інфекцію, що передається через кров. Яке з перерахованих захворювань найменш імовірно передається через кров?

SLE SLE

AIDS СНІД

HCV HCV

HBV HBV

HIV ВІЛ

155 / 180
A 38-year-old woman, who was diagnosed with systemic lupus erythematosus (SLE) 3 years ago, comes to her physician with a complaint of facial swelling and decreased urination that she first noticed 2 weeks ago. She currently takes azathioprine and a corticosteroid. Her vital signs show blood pressure - 150/90 mm Hg, pulse - 91/min., temperature - 36.8°С and respiratory rate - 15/min. On physical examination, the doctor notices erythematous rash on her face exhibiting a butterfly pattern. The laboratory studies reveal hypercholesterolemia, hypertriglyceridemia and proteinuria. Which of the following is the most likely mechanism of SLE’s complication in this patient? 38-річна жінка, якій 3 роки тому поставили діагноз системний червоний вовчак (СЧВ), звернулася до лікаря зі скаргою на набряк обличчя та зменшення сечовипускання, яке вперше помітила 2 тижні тому, зараз приймає азатіоприн і кортикостероїд, за життєвими показаннями артеріальний тиск - 150/90 мм рт.ст., пульс - 91/хв., температура - 36,8°С, частота дихання - 15/хв. .Під час фізикального огляду лікар помічає еритематозний висип на обличчі з малюнком метелика. Лабораторні дослідження виявляють гіперхолестеринемію, гіпертригліцеридемію та протеїнурію. Що з перерахованого є найбільш імовірним механізмом ускладнення СЧВ у цієї пацієнтки?

Decrease in renal blood flow (ischemic nephropathy) Зниження ниркового кровотоку (ішемічна нефропатія)

Acute infection of the kidney Гостра інфекція нирок

Increased plasma oncotic pressure Підвищений онкотичний тиск плазми

Immune complex-mediated glomerular disease клубочкова хвороба, опосередкована імунними комплексами

156 / 180
A 24-year-old man undergoes a surgery and during the operation, an organ is excised and sent for histological evaluation. A light microscopic examination reveals the organ encased by thin connective tissue capsule that enters the substance of the lobes to further subdivide the organ into irregular lobular units. Each lobule contains a cluster of follicles filled with colloid. Follicular epithelium consists of low columnar, cuboidal or squamous cells, depending on the level of activity of the follicle. Which of the following organs does this tissue most likely belong to? 24-річному чоловікові роблять операцію, і під час операції орган вирізають і відправляють на гістологічне дослідження. Світлове мікроскопічне дослідження виявляє, що орган закритий тонкою сполучнотканинною капсулою, яка входить у речовину часточок, щоб далі поділити орган на неправильні часточкові одиниці. Кожна часточка містить скупчення фолікулів, заповнених колоїдом. Фолікулярний епітелій складається з низьких стовпчастих, кубовидних або плоских клітин, залежно від рівня активності фолікула. До якого з наведених органів найімовірніше належить ця тканина?

Pancreas Підшлункова залоза

Parathyroid gland Паращитовидна залоза

Parotid gland Привушна залоза

Thyroid gland Щитовидна залоза

Thymus Тимус

157 / 180
A researcher is investigating the relationship between inflammatory mediators. He performs an experiment, investigating the effect of nonsteroidal anti-inflammatory drugs (NSAIDs) on patients with high-grade fever. His research indicates that certain NSAIDs act as competitively reversible inhibitors of the cyclooxygenase (COX) enzymes. It is known that COX catalyzes the formation of prostaglandins from a certain molecule that itself is derived from the cellular phospholipid bilayer by phospholipase A2. Which of the following molecules is a precursor of an inflammatory mediators mentioned above? Дослідник досліджує взаємозв'язок між медіаторами запалення. Він проводить експеримент, досліджуючи вплив нестероїдних протизапальних препаратів (НПЗП) на пацієнтів з високою формою Його дослідження вказують на те, що певні НПЗП діють як конкурентні оборотні інгібітори ферментів циклооксигенази (ЦОГ). Відомо, що ЦОГ каталізує утворення простагландинів із певної молекули, яка сама походить від клітинного фосфоліпідного подвійного шару фосфоліпазою А2. наступні молекули є попередниками медіаторів запалення, згаданих вище?

Palmitic acid Пальмітинова кислота

Arachidonic acid Proopiomelanocortin Проопіомеланокортин арахідонової кислоти

Tyrosine Тирозин

Cholesterol Холестерин

158 / 180
A medical student studies a waste disposal system in human epithelial cells. During electronic microscopy he reveals the spherical vesicles, surrounded by a membrane and containing many different hydrolytic enzymes. The main function of these organelles is to provide intracellular digestion and protective reactions of the cell. Which of the following organelles is mentioned above? Студент-медик вивчає систему утилізації відходів в епітеліальних клітинах людини. Під час електронної мікроскопії він виявляє сферичні везикули, оточені мембраною та містять багато різних гідролітичних ферментів. Основною функцією цих органел є забезпечення внутрішньоклітинного травлення та захисних реакцій клітини.Яка з наведених органел згадана вище?

Endoplasmatic reticulum Ендоплазматичний ретикулум

Ribosomes Рибосоми

Lysosomes Лізосоми

Mitochondria Мітохондрії

Centrosomes Центросоми

159 / 180
A biology graduate student is performing an experiment in the immunology laboratory. He studies a blood cell count from a patient with acute appendicitis, which shows an increase in the number of cells having a multilobed nucleus and multiple cytoplasmic granules. These cells engulf pathogens or necrotic tissue and help in the degradation of foreign products. Which of the following procescsses is seen in the cell desbribed above? Аспірант біології проводить експеримент в імунологічній лабораторії. Він вивчає кількість клітин крові пацієнта з гострим апендицитом, що показує збільшення кількості клітин, що мають багаточасткове ядро та численні цитоплазматичні гранули. Ці клітини поглинають патогени або некротичну тканину та допомагають у деградації чужорідних продуктів. Який із наведених нижче процесів спостерігається в клітині, описаній вище?

Parietal digestion Пристінкове травлення

Passive diffusion Пасивна дифузія

Pinocytosis Піноцитоз

Osmosis Осмос

Phagocytosis Фагоцитоз

160 / 180
A patient suffers from a condition which, is characterized by a restriction in blood supply to tissues which leads to inadequate oxygen delivery to cells and contravention of cell metabolism. It is often caused by partial or total blockage of arteries. Which of the following is developed in this patient? Пацієнт страждає на стан, який характеризується обмеженням кровопостачання тканин, що призводить до недостатнього надходження кисню до клітин і порушення клітинного метаболізму. Це часто спричинено частковою або повною закупоркою артерій. Що з перерахованого нижче розвивається у цього пацієнта?

Inflammation Запалення

Hypoxia Гіпоксія

Embolism Емболія

Ischemia Ішемія

Spasm Спазм

161 / 180
A bone marrow biopsy of an 8-year-old girl shows a group of cells which has undergone the process of pyknosis and loss of the nucleus during its differentiation. Which of the following types of hemopoiesis is characterised by the morphological changes described above? Біопсія кісткового мозку 8-річної дівчинки показує групу клітин, яка зазнала процесу пікнозу та втрати ядра під час диференціювання. Для якого з наведених типів кровотворення характерні описані вище морфологічні зміни?

Erytropoesis Еритропоез

Lymphocytopoesis Thrombocytopoesis Лімфоцитопоез Тромбоцитопоез

Granulocytopoesis Гранулоцитопоез

Monocytopoesis Моноцитопоез

162 / 180
The healthcare provider performs a complete blood count to find out if the bone marrow is making the right number of blood cells. He studies a blood cell that has no nucleus and has a function to react to bleeding of an injured blood vessel by clumping, thereby initiating a blood clot. Which of the following is the main object of testing? Лікар проводить повний аналіз крові, щоб з’ясувати, чи кістковий мозок виробляє правильну кількість клітин крові. Він вивчає клітину крові, яка не має ядра і має функцію реагувати на кровотечу з ушкодженої кровоносної судини шляхом злипання, тим самим ініціюючи тромб. Що з наведеного нижче є основним об’єктом тестування?

Leukocytes Лейкоцити

Macrophages Макрофаги

Platelets Stem cells Стовбурові клітини тромбоцитів

Prothrombine Протромбін

163 / 180
With total starvation the only source of water for the body is the oxidation process of organic compounds. Which of the following substances under these conditions is the main source of endogenic water? При повному голодуванні єдиним джерелом води для організму є процес окислення органічних сполук. Яка з перерахованих речовин за цих умов є основним джерелом ендогенного вода?

Glycoproteins Глікопротеїни

Lipids Ліпіди

Carbohydrates Вуглеводи

Proteins Білки

Lipoproteins Ліпопротеїни

164 / 180
An 18-year-old girl comes to her physician with concern about her health because she has not achieved menarche. She denies any significant weight loss, changes in mood, or changes in her appetite. She mentions that her mother told her about mild birth defects, but she cannot recall the specifics. Past medical history and family history are benign. On physical examination, the patient is short in stature, has a short and webbed neck and wide chest. Staining of buccal smear reveals absence of Barr bodies in the nucleis of epithelial cells. A urine pregnancy test is negative. Which of the following genetic disorders is the most likely cause of this patient’s condition? 18-річна дівчина звернулася до свого лікаря із занепокоєнням щодо свого здоров’я, оскільки у неї не настала менархе. Вона заперечує будь-яку значну втрату ваги, зміни настрою , або зміни апетиту. Вона згадує, що мати розповідала їй про легкі вроджені дефекти, але вона не може пригадати подробиць. Історія хвороби та сімейний анамнез є доброякісними. Під час медичного обстеження пацієнтка низького зросту, має низький і перетинчаста шия та широка грудна клітка Фарбування буккального мазка виявляє відсутність тілець Барра в ядрах епітеліальних клітин Тест сечі на вагітність негативний Яке з наступних генетичних захворювань є найбільш імовірною причиною стану даної пацієнтки?

Cri du chat ('cat-cry') syndrome Синдром крику кота

Klinefelter syndrome Синдром Клайнфельтера

Edwards syndrome Синдром Едвардса

Patau syndrome Синдром Патау

Turner syndrome Синдром Тернера

165 / 180
A 6-year-old boy is brought to the pediatrician by his mother, who complains of low-grade fever, chronic cough and night sweats in her child. She describes the cough as productive, producing white sputum that is sometimes streaked with blood. She also says that her son has lost some weight in the last month. His vital signs include blood pressure of 115/75 mm Hg, heart rate of 110/min., respiratory rate of 18/min. and temperature of 36.6°On physical examination, the patient is ill looking. Pulmonary auscultation reveals some fine crackles in the right upper lobe. The pediatrician suspects an active infection and performs Mantoux test. Intradermal injection of which of the following substances has been most likely used by pediatrician for the screening test in this clinical case? Мати привела хлопчика 6 років до педіатра, яка скаржиться на субфебрильну температуру, хронічний кашель і нічну пітливість. Вона описує кашель як продуктивний, з виділенням білого мокротиння, іноді з прожилками крові.Вона також каже, що її син трохи схуд за останній місяць.Його життєві показники включають артеріальний тиск 115/75 мм рт.ст., пульс 110/. хв., частота дихання 18/хв., температура 36,6°. При фізикальному огляді хворий виглядає погано.Під час аускультації легенів у правій верхній частці виявляються дрібні хрипи.Педіатр підозрює активну інфекцію та проводить пробу Манту.Внутрішньошкірна ін’єкція. яка з наведених нижче речовин, швидше за все, була використана педіатром для скринінгового тесту в цьому клінічному випадку?

Tetanus and diphtheria toxoids vaccine (Td) Вакцина проти правцевого та дифтерійного анатоксинів (Td)

Bacillus Calmette-Guerin (BCG) vaccine Вакцина проти бацили Кальмета-Герена (БЦЖ)

Tuberculin Туберкулін

Diphtheria-tetanus toxoids-acellular pertussis vaccine (DTaP) Дифтерійно-правцевий анатоксини безклітинна кашлюкова вакцина (DTaP)

166 / 180
A research group is investigating a complex of three enzymes. They have created cultures of myocytes derived from high-performance college athletes and simulated starvation conditions. After the experiment they concluded that during starvation the amount of this complex in the muscle tissue was higher. The complex converts pyruvate into acetyl-coenzyme-which enters the citric acid cycle (Krebs cycle) under aerobic conditions. This reaction also involves the further reduction of NAD+ molecules into NADH. An activating effect of which of the following enzymes is described above? Дослідницька група досліджує комплекс із трьох ферментів. Вони створили культури міоцитів, отриманих від високопродуктивних спортсменів коледжу, і змоделювали умови голодування. Після експерименту вони прийшли до висновку, що під час голодування кількість цього комплексу в м'язовій тканині була більшою.Комплекс перетворює піруват в ацетилкофермент, який в аеробних умовах вступає в цикл лимонної кислоти (цикл Кребса).Ця реакція також включає подальше відновлення молекул NAD+ до NADH Активуюча дія якого з ферментів описана вище?

Pyruvate dehydrogenase Піруватдегідрогеназа

Lactate dehydrogenase Лактатдегідрогеназа

Phosphofructokinase Фосфофруктокіназа

Hexokinase Гексокіназа

Phosphorylase Фосфорилаза

167 / 180
This action is an involuntary and nearly instantaneous movement in response to a stimulus. It is made possible by neural pathways which can act on an impulse before that impulse reaches the brain and does not require conscious thought. Which of the following actions is described above? Ця дія є мимовільним і майже миттєвим рухом у відповідь на подразник. Це стає можливим завдяки нейронним шляхам, які можуть діяти на імпульс до того, як цей імпульс досягне мозок і не вимагає свідомого мислення. Яка з наступних дій описана вище?

Reflex Рефлекс

Initiation Початок

Tetanus Правець

Neuralgia Невралгія

Defense Захист

168 / 180
A 16-year-old girl concerned about her sexual development comes to the physician. She mentions that she has still not had a menstrual period. However, she is otherwise a healthy girl with no significant, medical problems since birth. On physical examination, her vital signs are stable. She does not have pubic hair and her breast is slightly elevated with areola remaining in contour with surrounding breast. Which of the following is the most likely cause of this abnormal physical development? 16-річна дівчина, стурбована своїм статевим розвитком, приходить до лікаря. Вона згадує, що у неї досі не було менструації. Однак вона в іншому випадку здорова дівчинка без значних медичних проблем з моменту народження. Під час фізичного обстеження її життєво важливі показники стабільні. У неї немає волосся на лобку, її груди трохи підняті, ареола зберігає контур навколо грудей. Що з наведеного нижче найімовірнішою причиною цього відхилення у фізичному розвитку?

Ovarian insufficiency Недостатність яєчників

Hypothyroidism Гіпотиреоз

Hyperthyroidism Гіпертиреоз

Pancreatic islet insufficiency Недостатність острівців підшлункової залози

Adrenal medulla hyperfunction Гіперфункція мозкової речовини надниркових залоз

169 / 180
A 28-year-old male patient presents to the physician with pain and discomfort in his right lower extremity. On physical exam, palpation of the patient’s pulses reveals the absence of pulsation in the dorsalis pedis and tibialis posterior arteries, while the pulsation on the femoral artery is present. The skin of his extremity is also pale and cold/ Which of the following arteries is most likely damaged in this patient? Пацієнт віком 28 років звернувся до лікаря з болем і дискомфортом у правій нижній кінцівці. Під час медичного огляду пальпація пульсу пацієнта виявляє відсутність пульсації на задній артерії dorsalis pedis і tibialis posterior, а пульсація на стегновій артерії присутня Шкіра кінцівки також бліда і холодна/ Яка з перерахованих артерій найбільш імовірно пошкоджена у даного хворого?

External iliac artery Зовнішня клубова артерія

Popliteal artery Підколінна артерія

Tibial artery Гомілкова артерія

Descending genicular artery Спадна колінчаста артерія

Deep femoral artery Глибока стегнова артерія

170 / 180
A pathologist studies a specimen of the small bean-shaped structure which is the part of human immune system. In a cross section it consists of an outer layer (cortex) and inner layer (medulla), and is surrounded by a fibrous capsule and subscapular sinus and is about 1.8 cm long. Which of the following is being studied by the pathologist? Патолог вивчає зразок невеликої бобоподібної структури, яка є частиною імунної системи людини. У поперечному розрізі він складається із зовнішнього шару (кори ) і внутрішній шар (medulla), і оточений фіброзною капсулою та підлопатковим синусом і має довжину приблизно 1,8 см. Що з наведеного нижче вивчає патолог?

Salivary gland Слинна залоза

Lymph node Лімфатичний вузол

Thymus Тимус

Spleen Сплін

Parathyroid gland Паращитовидна залоза

171 / 180
A 27-year-old female presents with a severe sore throat, hoarseness, painful swallowing and low-grade fever. On intraoral examination, a large grey membrane is noticed on the oropharynx. Removal of the membrane reveals a bleeding oedematous mucosa. Which of the following is the most likely diagnosis? У 27-річної жінки сильний біль у горлі, захриплість, біль при ковтанні та субфебрильна температура. Під час внутрішньоротового огляду велика сіра оболонка помічено на ротоглотці. Видалення оболонки виявляє кровоточиву набряклу слизову оболонку. Що з перерахованого є найбільш імовірним діагнозом?

Measles Кір

Streptococcal pharyngitis Стрептококовий фарингіт

Scarlet fever Скарлатина

Diphtheria Дифтерія

Meningococcal disease Менінгококова інфекція

172 / 180
The doctor evaluates his patient’s spyrography. One of the evaluation parameters represents the normal amount of air displaced between normal inhalation and exhalation without any extra efforts or appointments. Which of the following is being evaluated in this case? Лікар оцінює спірографію свого пацієнта. Один із параметрів оцінки являє собою нормальну кількість повітря, що витісняється між нормальним вдихом і видихом без будь-яких додаткових зусиль або призначень. Який із у цій справі оцінюється таке?

Inspiratory capacity Ємність вдиху

Total lung capacity Загальна ємність легень

Residual volume Залишковий об'єм

Tidal volume Дихальний об'єм

Vital volume Життєвий об'єм

173 / 180
A 37-year-old man is admitted to a hospital with mental confusion and disorientation. His wife reports he became more irritable and forgetful in the past year. In addition, she notes that he became a vegan a year ago, and currently, his diet consists of starchy foods like potatoes, corn, and leafy vegetables. GI symptoms include anorexia, diarrhea and vomiting. He has glossitis and skin lesions that appear as vesicles over the extremities. Eczema-like lesions around the mouth, as well as desquamation and roughened skin over the hands, are also present. Neurologic examination reveals symmetrical hypesthesia for all types of sensation in both upper and lower extremities in a 'gloves and socks' distribution. Deficiency in the diet the of which of the following amino acids is the most likely cause of this condition? 37-річний чоловік потрапив до лікарні зі сплутаністю свідомості та дезорієнтацією. Його дружина повідомляє, що останній рік він став більш дратівливим і забудькуватим. У Крім того, вона зазначає, що він став веганом рік тому, і наразі його дієта складається з крохмалистих продуктів, таких як картопля, кукурудза та листові овочі.Шлунково-кишкові симптоми включають анорексію, діарею та блювання.У нього глосит та ураження шкіри, які виглядають як пухирці на кінцівках.Екземоподібні ураження навколо рота, а також лущення та огрубіння шкіри на руках також присутні.Неврологічний огляд виявляє симетричну гіпестезію для всіх типів відчуття у верхніх і нижніх кінцівках у «рукавичках і шкарпетках» розподіл.Дефіцит якої з наступних амінокислот є найбільш імовірною причиною цього стану?

Lysine Лізин

Histidine Гистидин

Arginine Аргінін

Threonine Треонін

Tryptophan Триптофан

174 / 180
The main function of the human glands are to produce and release substances that perform a specificfunction in the body. According to the classification there are endocrine and exocrine glands. But also there are glands that may be classified as both. Which of the following glands can be endo-and exocrine simultaneously? Основною функцією залоз людини є виробництво та вивільнення речовин, які виконують певну функцію в організмі. Відповідно до класифікації є ендокринні та екзокринні залози. Але також є залози, які можна класифікувати як обидві. Яка з наведених нижче залоз може бути ендо- та екзокринною одночасно?

Gastrointestinal Шлунково-кишковий тракт

Sebaceous Сальні

Salivary Слина

Parathyroid Паращитовидна залоза

Lacrimal Сльозотеча

175 / 180
Physiological solution is an isotonic diluent used to maintain cell integrity and viability in procedures that -require preparation of test suspension of organisms. This sterile solution in water provides osmotic protection for microbial cells. Which of the following concentrations of NaCl is considered to be physiological? Фізіологічний розчин - це ізотонічний розчинник, який використовується для підтримки цілісності та життєздатності клітин у процедурах, які вимагають приготування тестової суспензії організмів. Цей стерильний розчин у воді забезпечує осмотичний захист для мікробних клітин.Яка з наведених концентрацій NaCl вважається фізіологічною?

5% 5%

10% 10%

0.9% 0,9%

9% 9%

9% 9%

176 / 180
A 60-year-old man with a history of hypertension, diabetes and hyperlipidemia had a sudden onset of right-sided weakness. By the time the ambulance arrived, he had difficulty speaking. Unfortunately, the patient died within the next 2 hours and an autopsy was performed immediately. The gross examination of the cerebral left hemisphere showed brain swelling, widened gyri and poorly demarcated gray-white junction. Which of the following is the most likely cause of this patient’s death? У 60-річного чоловіка з гіпертензією, діабетом і гіперліпідемією раптово виникла слабкість правого боку. На момент прибуття швидкої допомоги йому було важко говорити. На жаль, пацієнт помер протягом наступних 2 годин, і негайно було проведено розтин. Загальний огляд лівої півкулі головного мозку показав набряк мозку, розширені звивини та погано розмежований сіро-білий з’єднання. Що з наступного є найімовірнішою причиною смерті цього пацієнта?

Abscess Абсцес

Cyst Кіста

Tumor Пухлина

Intracerebral hemorrhage Внутрішньомозковий крововилив

Ischemic stroke Ішемічний інсульт

177 / 180
A 14-year-old girl presents to the emergency department for evaluation of an 'infected leg'. She states there is no history of trauma but mentions she had a history of sickle cell disease/ On physical examination, her upper part of the right shin is very painful, red, swollen, and hot. Her temperature is 39.2°An X-ray shows focal bony lysis and loss of trabecular architecture in the metaphysis of the right tibiIncreased activity of which of the following cells is the most likely cause of bone reabsorption in this patient? 14-річна дівчина звертається до відділення невідкладної допомоги для оцінки «інфікованої ноги». Вона стверджує, що в анамнезі немає травми, але згадує, що мала Серповидно-клітинна анемія в анамнезі/ Під час фізикального обстеження її верхня частина правої гомілки дуже болить, червона, набрякла та гаряча Її температура 39,2°. Рентгенівський знімок показує вогнищевий лізис кістки та втрату трабекулярної архітектури в метафізі правої великогомілкової кістки. Підвищена активність якої з наступних клітин є найбільш імовірною причиною реабсорбції кісткової тканини у цього пацієнта?

Osteoblasts Остеобласти

Chondrocytes Хондроцити

Osteoclasts Остеокласти

Chondroblasts Хондробласти

Osteocytes Остеоцити

178 / 180
A 46-year-old man presents with fatigue and joint pain in his fingers and wrists for the last 2 months. The pain is present in both hands and the wrists are swollen. Furthermore, he describes morning stiffness in his joints lasting about 2 hours, which improves with use. His past medical history reveals he has been successfully treated for H. pylori related ulcers last year. He denies smoking and stopped drinking when his gastric symptoms started. Which of the following drugs is the best choice for his joints’ pain management? 46-річний чоловік відчуває втому та біль у суглобах у пальцях і зап’ястях протягом останніх 2 місяців. Біль присутній в обох руках і зап'ястя опухлі. Крім того, він описує ранкову скутість у суглобах, яка триває приблизно 2 години, яка покращується під час використання. Його минула медична історія показує, що минулого року він успішно лікувався від виразки, пов'язаної з H. pylori. Він заперечує куріння та кинув пити, коли його почалися шлункові симптоми. Який із наведених препаратів є найкращим вибором для лікування болю в його суглобах?

Aspirin Аспірин

Paracetamol Парацетамол

Morphine Морфін

Celecoxib Целекоксиб

Prednisone Преднізон

179 / 180
A 20-year-old female comes to the clinic after missing her last 2 periods. Her cycles are usually regular, occurring at 28-30 day interval with moderate bleeding and some abdominal discomfort. She also complains of progressively diminishing peripheral vision. Her doctor reveals loss of vision in the lateral halves of both eyes. Involvement of which of the following structures would you most likely expect to be the reason of bitemporal hemianopsia? 20-річна жінка прийшла в клініку після відсутності останніх 2 менструацій. Її цикли зазвичай регулярні, відбуваються з інтервалом 28-30 днів з помірним кровотеча та певний дискомфорт у животі. Вона також скаржиться на прогресуюче погіршення периферичного зору. Її лікар виявляє втрату зору в бічних половинах обох очей. Ураження якої з наступних структур, на вашу думку, є причиною бітемпоральної геміанопсії?

Right optic tract Правий зоровий тракт

Left optic nerve Лівий зоровий нерв

Left optic tract Лівий зоровий тракт

Optic chiasm Оптичний хіазм

Right optic nerve Правий зоровий нерв

180 / 180
A 25-year-old male presents to an emergency department with suddenonset right-sided pleuritic chest pain and breathlessness. On examination, he has arterial blood saturation of 90% in air and is afebrile. An X- ray shows a lung edge visible in the right hemithorax, beyond which no lung markings are seen. Which of the following conditions has most likely occured in this patient? 25-річний чоловік звернувся до відділення невідкладної допомоги з раптово виниклим правостороннім плевритним болем у грудній клітці та задишкою. Під час огляду він має сатурацію артеріальної крові 90 % у повітрі та без лихоманки. Рентгенівський знімок показує край легені, видимий у правій гемітораксі, за межами якого не видно жодних легеневих позначок. Яке з наведених нижче станів найімовірніше виникло у цього пацієнта?

Rib fracture Перелом ребра

Pleurisy Плеврит

Aerophagy Аерофагія

Pneumonia Пневмонія

Pneumothorax Пневмоторакс